ChaseDream

标题: 【每日逻辑练习第二季】【1-15】 [打印本页]

作者: fox0923    时间: 2011-12-3 12:26
标题: 【每日逻辑练习第二季】【1-15】
到我了,由于晚些时候可能不一定能准时贴上来,所以先把转天的贴了,大家不要图快做,稳扎稳打~~~

精练

2. Most antidepressant drugs cause weight gain. While
dieting can help reduce the amount of weight gained
while taking such antidepressants, some weight gain
is unlikely to be preventable.
The information above most strongly supports which
one of the following?


(A) A physician should not prescribe any
antidepressant drug for a patient if that
patient is overweight.
(B) People who are trying to lose weight should not
ask their doctors for an antidepressant drug.
(C) At least some patients taking antidepressant
drugs gain weight as a result of taking them.
(D) The weight gain experienced by patients taking
antidepressant drugs should be attributed to
lack of dieting.
(E) All patients taking antidepressant drugs should
diet to maintain their weight.




PREP 练习


57.    (34655-!-item-!-188;#058&007580)


Roland:  The alarming fact is that 90 percent of the people in this country now report that they know someone who is unemployed.


Sharon:  But a normal, moderate level of unemployment is 5 percent, with 1 out of 20 workers unemployed.  So at any given time if a person knows approximately 50 workers, 1 or more will very likely be unemployed.


Sharon's argument is structured to lead to which of the following as a conclusion?


(A) The fact that 90% of the people know someone who is unemployed is not an indication that unemployment is abnormally high.
(B) The current level of unemployment is not moderate.
(C) If at least 5% of workers are unemployed, the result of questioning a representative group of people cannot be the percentage Roland cites.
(D) It is unlikely that the people whose statements Roland cites are giving accurate reports.
(E) If an unemployment figure is given as a certain percent, the actual percentage of those without jobs is even higher.


58.    (34705-!-item-!-188;#058&007581)


Roland:  The alarming fact is that 90 percent of the people in this country now report that they know someone who is unemployed.


Sharon:  But a normal, moderate level of unemployment is 5 percent, with 1 out of 20 workers unemployed.  So at any given time if a person knows approximately 50 workers, 1 or more will very likely be unemployed.


Sharon's argument relies on the assumption that


(A) normal levels of unemployment are rarely exceeded
(B) unemployment is not normally concentrated in geographically isolated segments of the population
(C) the number of people who each know someone who is unemployed is always higher than 90% of the population
(D) Roland is not consciously distorting the statistics he presents
(E) knowledge that a personal acquaintance is unemployed generates more fear of losing one's job than does knowledge of unemployment statistics


59.    (34753-!-item-!-188;#058&007583)


A certain mayor has proposed a fee of five dollars per day on private vehicles entering the city, claiming that the fee will alleviate the city's traffic congestion.  The mayor reasons that, since the fee will exceed the cost of round-trip bus fare from many nearby points, many people will switch from using their cars to using the bus.


Which of the following statements, if true, provides the best evidence that the mayor's reasoning is flawed?


(A) Projected increases in the price of gasoline will increase the cost of taking a private vehicle into the city.
(B) The cost of parking fees already makes it considerably more expensive for most people to take a private vehicle into the city than to take a bus.
(C) Most of the people currently riding the bus do not own private vehicles.
(D) Many commuters opposing the mayor's plan have indicated that they would rather endure traffic congestion than pay a five-dollar-per day fee.
(E) During the average workday, private vehicles owned and operated by people living within the city account for twenty percent of the city's traffic congestion.


60.    (34801-!-item-!-188;#058&007584)


Because no employee wants to be associated with bad news in the eyes of a superior, information about serious problems at lower levels is progressively softened and distorted as it goes up each step in the management hierarchy.  The chief executive is, therefore, less well informed about problems at lower levels than are his or her subordinates at those levels.


The conclusion drawn above is based on the assumption that


(A) problems should be solved at the level in the management hierarchy at which they occur
(B) employees should be rewarded for accurately reporting problems to their superiors
(C) problem-solving ability is more important at higher levels than it is at lower levels of the management hierarchy
(D) chief executives obtain information about problems at lower levels from no source other than their subordinates
(E) some employees are more concerned about truth than about the way they are perceived by their superiors





作者: fox0923    时间: 2011-12-3 12:30
精练:
Question #2. Must-CE. December 2001 LSAT, Section 1, #4. The correct answer choice is (C)
The causal relationship in this problem appears in the premise, and the argument is structured as follows:
Premise: Most antidepressant drugs cause weight gain.
Premise: Dieting can help reduce the amount of weight gained while taking such antidepressants
Conclusion: Some weight gain is unlikely to be preventable.
Note that the causal premise specifically states that “most” antidepressants cause weight gain, not
necessarily all antidepressants. Also, the second premise specifically refers to antidepressants causing
weight gain (the use of “such” indicates this). The second premise also indicates that the amount gained
can be reduced, not that dieting can stop weight gain. Perhaps the antidepressants cause a twenty pound
weight gain, but dieting can reduce that to a ten pound total gain.
The question stem is a Must Be True, and thus you must accept the stimulus information and find an
answer that is proven by that information.
Answer choice (A): This is an Exaggerated answer. The stimulus indicates that most antidepressants cause
weight gain, leaving open the possibility that some do not. This answer choice references any
antidepressant drug. Further, the stimulus does not address the role of a physician or the advisability of
prescribing certain drugs under certain conditions. The benefits of prescribing an antidepressant that causes
weight gain to an overweight patient may well outweigh the negatives (pun intended).
Answer choice (B): This is also an Exaggerated answer. The stimulus allows for antidepressants that do
not cause weight gain.
Answer choice (C): This is the correct answer. Some individuals taking antidepressants that cause weight
gain will gain weight even though dieting can reduce the amount of the gain.
Answer choice (D): This is an Opposite answer. The stimulus and correct answer both indicate that people
taking the weight gain-causing antidepressants will gain weight regardless of whether they diet. Thus, the
weight gain cannot be attributed to a lack of dieting.
Answer choice (E): This answer is too strong. Not all patients necessarily take antidepressants that cause
weight gain, so those that do not might not need to diet to maintain their weight.


PREP Key:ABBD
作者: qiuhua01234567    时间: 2011-12-3 12:56
沙发,哈哈哈
Support

Background: Most antidepressant drugs cause weight gain.

Premise: Whiledieting can help reduce the amount of weight gainedwhile taking such antidepressants

Conclusion: some weight gainis unlikely to be preventable.

(A)A physician should not prescribe any
antidepressant drug for a patient if that
patient is overweight.

----------------------------------------------------------------------irrelevent
(B) People who are trying to lose weight should not
ask their doctors for an antidepressant drug.

----------------------------------------------------------------------irrelevent
(C) At least some patients taking anti depressantdrugs gain weight as a result of taking them.

---------------------------------------------------------------------------------correct
(D) The weight gain experienced by patients takingantidepressant drugs should be attributed tolack of dieting.

-----------------------------------------------------------------------irrelevent
(E) All patients taking antidepressant drugs shoulddiet to maintain their weight.
------------------------------------------------------------------------------------support

(A)The fact that 90% of the people know someone who is unemployed is not an indication that unemployment is abnormally high.

-------------------------------------------------------------------------------correct.
(B) The current level of unemployment is not moderate.
(C) If at least 5% of workers are unemployed, the result of questioning a representative group of people cannot be the percentage Roland cites.
(D) It is unlikely that the people whose statements Roland cites are giving accurate reports.
(E) If an unemployment figure is given as a certain percent, the actual percentage of those without jobs is even higher.

(A)normal levels of unemployment are rarely exceeded
(B) unemployment is not normally concentrated in geographically isolated segments of the population
(C) the number of people who each know someone who is unemployed is always higher than 90% of the population

-----------------------------------------------------------------------------correct
(D) Roland is not consciously distorting the statistics he presents
(E) knowledge that a personal acquaintance is unemployed generates more fear of losing one's job than does knowledge of unemployment statistics
background:
A certain mayor has proposed a fee of five dollars per day on private vehicles entering the city, claiming that the fee will alleviate the city's traffic congestion.

Premise: the fee will exceed the cost of round-trip bus fare from many nearby points

Conclusion: many people will switch from using their cars to using the bus.

(A)Projected increases in the price of gasoline will increase the cost of taking a private vehicle into the city.
(B) The cost of parking fees already makes it considerably more expensive for most people to take a private vehicle into the city than to take a bus.

------------------------------------------------------------------------------------------correct
(C) Most of the people currently riding the bus do not own private vehicles.
(D) Many commuters opposing the mayor's plan have indicated that they would rather endure traffic congestion than pay a five-dollar-per day fee.
(E) During the average workday, private vehicles owned and operated by people living within the city account for twenty percent of the city's traffic congestion.
我为什么选的是C

Assumption

Premiseno employee wants to be associated with bad news in the eyes of a superior, information about serious problems at lower levels is progressively softened and distorted as it goes up each step in the management hierarchy

ConclusionThe chief executive is, therefore, less well informed about problems at lower levels than are his or her subordinates at those levels.

(A)problems should be solved at the level in the management hierarchy at which they occur

(B)employees should be rewarded for accurately reporting problems to their superiors
(C) problem-solving ability is more important at higher levels than it is at lower levels of the management hierarchy
(D) chief executives obtain information about problems at lower levels from no source other than their subordinates

--------------------------------------------------------------------------------------------------correct
(E) some employees are more concerned about truth than about the way they are perceived by their superiors






作者: fox0923    时间: 2011-12-3 13:02
我也来了,排队,排队~~

----------------------------------------------------------------------------------
精炼---------------------22s-----------------------Main Conclusion
P: The antidepressant drugs can cause weight gain.
P: However, the dieting can reduce the amount of weight gain while taking antidepressant drugs.
C: Therefore, there would be some weight gain.
Prephrase: The weight gain cannot be completely prevented by dieting only.
Analysis:

(A) A physician should not prescribe any
antidepressant drug for a patient if that
patient is overweight.-------------------------------------There's no where has ever mentioned "physician" should prescribe drugs.
(B) People who are trying to lose weight should not
ask their doctors for an antidepressant drug.---------The consultation of drugs from doctors is not the main point of this argument.
(C) At least some patients taking antidepressant
drugs gain weight as a result of taking them.---------yeah, this conclusion summarizes premise 1 and 2.
(D) The weight gain experienced by patients taking
antidepressant drugs should be attributed to
lack of dieting.---------------------------------------------we cannot conclude this because the passage does not state the control level of dieting.
(E) All patients taking antidepressant drugs should
diet to maintain their weight.----------------------------diet is just one way to reduce the weight gain, but the passage doesn't state such way is the only method.




哇咔,作业完成~~
作者: UlysessHope    时间: 2011-12-3 14:08
狐狐姐~~~
作者: xeyyxzty    时间: 2011-12-3 14:53
1. premise:Most antidepressant drugs cause weight gain. dieting can help reduce the amount of weight gained while taking such antidepressantsconclusion:some weight gain is unlikely to be preventable.
???
(A) A physician should not prescribe any antidepressant drug for a patient if that patient is overweight.
--irrelevant
(B) People who are trying to lose weight should not ask their doctors for an antidepressant drug.
--support? or irrelevant
(C) At least some patients taking antidepressant drugs gain weight as a result of taking them.
--support--right
(D) The weight gain experienced by patients taking antidepressant drugs should be attributed to lack of dieting.
--weaken
(E) All patients taking antidepressant drugs should diet to maintain their weight.

--irrelevant
such 和most有联系~~~
Answer choice (A): This is an Exaggerated answer. The stimulus indicates that most antidepressants cause weight gain, leaving open the possibility that some do not. This answer choice references any antidepressant drug. Further, the stimulus does not address the role of a physician or the advisability of prescribing certain drugs under certain conditions. The benefits of prescribing an antidepressant that causes
weight gain to an overweight patient may well outweigh the negatives (pun intended).
Answer choice (B): This is also an Exaggerated answer. The stimulus allows for antidepressants that do
not cause weight gain.



作者: fox0923    时间: 2011-12-4 05:26
狐狐姐~~~
-- by 会员 UlysessHope (2011/12/3 14:08:21)



恩恩,hope回来了~拍瓜儿~~
作者: ugly5552000    时间: 2011-12-4 20:59
1/Background: most antidepressant drugs cause weight gain. Some weight gain can be reduced by dieting, while some is unpreventable.
Prephrase: dieting is not useful for every kind of weight gain.
C
A: this is an exaggerated answer, what if the advantages to the overweight patient brought by antidepressant drugs overweighs the negatives, even they are likely to cause weigh gain?
B: exaggerated answer, what if only antidepressant drug can cure their diseases?
C: right answer, dieting can help someone to lose weight, but someone not
D: irrelevant
E: diet sometimes is ineffective.
2/Background: 90percent of the people in the country report that someone they know has lost the job. The normal unemployment level is 5 percent, which means that if one knows about 50 workers, only 1 or more among them lost job is normal.
Prephrase: the level of unemployment in the country is very terrible.
A
A: Roland did not provide further information to support his argument.
B: Sharon’s argument begun with “but”, which showed she opposed Roland’s argument.
C: irrelevant
D: irrelevant
E: irrelevant
3/Background: 90percent of the people in the country report that someone they know has lost the job. The normal unemployment level is 5 percent, which means that if one knows about 50 workers, only 1 or more among them lost job is normal.
Prephrase: the questioned person and the workers they know do not know each other.
B
A: irrelevant
B: 90 percent persons might refer to same amount of unemployed workers
C: irrelevant
D: irrelevant
E: irrelevant
4/Background: a mayor proposed an extra fee per day on private vehicles entering the city and claimed that the proposal would ease the city’s congestion.
Premise: because of the extra fee, many drivers will switch their cars to using the bus.
Prephrase: most drivers’ destinations in the city cannot be reached by the buses.
B
A: inreferred information
B: the expensive parking cost and the extra fee will force most people to switch to bus.
C: irrelevant
D: irrelevant, no private vehicles, no extra fee
E: cannot be used as evidence.
5/Background: the fear of employees to be connected with bad news in the eyes of a superior makes information about serious problems at lower levels is progressively softened and distorted as it goes up each step in the management hierarchy.
Premise: the chief exexutive is less informed about problems at lower levels than are his or her subordinates at those levels.
Prephrase: the chief executive gets all information from his subordinates, and no one would snitch.
D
A: irrelevant
B: irrelevant
C: irrelevant
D: if chief executives obtain information only from his subordinates, he will surely be less informed.
E: irrelevant
作者: winghyy    时间: 2011-12-5 00:14
精炼题
25s - infer
antidepressant drugs cause weight gain. However, dieting helps reduce the amount of weight gained while taking drug. Therefore, some weight gain is unlikely to be preventable.
Prephase: the weight gained by the effect of drug is more than that reduced by dieting when taking drugs.
选C
(A) A physician should not prescribe any antidepressant drug for a patient if that patient is overweight.
——out of scope
(B) People who are trying to lose weight should not ask their doctors for an antidepressant drug.
——out of scope
(C) At least some patients taking antidepressant drugs gain weight as a result of taking them.
——correct  [At lease/some]
(D) The weight gain experienced by patients taking antidepressant drugs should be attributed to
lack of dieting.——the lack of dieting(wrong, contrary to the stimulus)
(E) All patients taking antidepressant drugs should diet to maintain their weight.
[all: extreme words]

看了解释,才发现自己的理解是多么的肤浅。。。限定词一定要注意
Note that the causal premise specifically states that “most” antidepressants cause weight gain, not
necessarily all
antidepressants. Also, the second premise specifically refers to antidepressants causing
weight gain (the use of “such” indicates this). The second premise also indicates that the amount gained
can be reduced, not that dieting can stop weight gain
..
作者: winghyy    时间: 2011-12-5 00:20
逻辑链
占位~补
作者: 风无衣    时间: 2011-12-5 12:49
【精炼1-15】
support_31s
premise:most  antidepressant drugs cause weight gain
premise:dieting could reduce the amout of weight gain while taking these drugs
conclusion:some weight gain is unlikely to preventable.
(A) A physician should not prescribe any
antidepressant drug for a patient if that
patient is overweight.——irrelevant
(B) People who are trying to lose weight should not
ask their doctors for an antidepressant drug.——irrelevant
(C) At least some patients taking antidepressant
drugs gain weight as a result of taking them.——correct
(D) The weight gain experienced by patients taking
antidepressant drugs should be attributed to
lack of dieting.—— people taking the weight gain-causing antidepressants will gain weight regardless of whether they diet. Thus, the weight gain cannot be attributed to a lack of dieting.
(E) All patients taking antidepressant drugs should
diet to maintain their weight.——some weight gain is unlikely to preventable.so it is highly impossible that all patients taking antidepressant drugs could maintain their weight by dieting.
作者: zz42050524    时间: 2011-12-5 15:02
26s  must be true
P: Antidepressant drug can cause weight gain. Dieting can help to reduce weight gain while taking the drugs, but some weight gain cannot be prevent.
理解: 最终意思是说增重是避免不了的,Diet只是辅助而不能完全解决问题。
(A) A physician should not prescribe any
antidepressant drug for a patient if that
patient is overweight.        Irrelevant
(B) People who are trying to lose weight should not
ask their doctors for an antidepressant drug.  有点那么个意思,这个要视病情而定了。
(C) At least some patients taking antidepressant
drugs gain weight as a result of taking them.   意思很大,就是C了
(D) The weight gain experienced by patients taking  这个和原文意思就不符了
antidepressant drugs should be attributed to
lack of dieting.
(E) All patients taking antidepressant drugs should   绝对了
diet to maintain their weight.
作者: balapupu    时间: 2011-12-5 16:38
1.[13s]
P: XX drugs cause weight gain? C: though diet can reduce the weight gain, the weight gain can not be prevented entirely.
Support conclusion: people want to lose weight should take some other measures.
(A) A physician should not prescribe any
antidepressant drug for a patient if that
patient is overweight.-->passage mention ”most”, however, the choice mention “any”, out of scope.
(B) People who are trying to lose weight should not
ask their doctors for an antidepressant drug.-? can not guarantee whether the people come across some specific situation.
(C) At least some patients taking antidepressant
drugs gain weight as a result of taking them.-->R
(D) The weight gain experienced by patients taking
antidepressant drugs should be attributed to
lack of dieting.-->the drugs should also be blamed.
(E) All patients taking antidepressant drugs should
diet to maintain their weight.-->it depends.
作者: xeyyxzty    时间: 2011-12-6 12:54
2.premise:Roland:  The alarming fact is that 90 percent of the people in this country now report that they know someone who is unemployed.
Sharon:  But a normal, moderate level of unemployment is 5 percent, with 1 out of 20 workers unemployed.  So at any given time if a person knows approximately 50 workers, 1 or more will very likely be unemployed.conclusion:就是90%都认识失业的人,这些人中有重复的
(A) The fact that 90% of the people know someone who is unemployed is not an indication that unemployment is abnormally high.
--right
(B) The current level of unemployment is not moderate.
--unknown
(C) If at least 5% of workers are unemployed, the result of questioning a representative group of people cannot be the percentage Roland cites.
--it can be
(D) It is unlikely that the people whose statements Roland cites are giving accurate reports.
--it is likely
(E) If an unemployment figure is given as a certain percent, the actual percentage of those without jobs is even higher.

--unknown
3.???
(A) normal levels of unemployment are rarely exceeded
--irrelevant
(B) unemployment is not normally concentrated in geographically isolated segments of the population
--right
(C) the number of people who each know someone who is unemployed is always higher than 90% of the population
--irrelevant
(D) Roland is not consciously distorting the statistics he presents
--irrelevant
(E) knowledge that a personal acquaintance is unemployed generates more fear of losing one's job than does knowledge of unemployment statistics
--irrelevant
4. background information:A certain mayor has proposed a fee of five dollars per day on private vehicles entering the city, claiming that the fee will alleviate the city's traffic congestion.
premise:the fee will exceed the cost of round-trip bus fare from many nearby points,
conclusion:many people will switch from using their cars to using the bus.
money is not the only cost for people--convenient, time-consuming ...
(A) Projected increases in the price of gasoline will increase the cost of taking a private vehicle into the city.
--irrelevant
(B) The cost of parking fees already makes it considerably more expensive for most people to take a private vehicle into the city than to take a bus.
--so there is hardly any effect of the new plan--right
(C) Most of the people currently riding the bus do not own private vehicles.
--irrelevant
(D) Many commuters opposing the mayor's plan have indicated that they would rather endure traffic congestion than pay a five-dollar-per day fee.
--support
(E) During the average workday, private vehicles owned and operated by people living within the city account for twenty percent of the city's traffic congestion.

--support
5. premise:Because no employee wants to be associated with bad news in the eyes of a superior, information about serious problems at lower levels is progressively softened and distorted as it goes up each step in the management hierarchy.
conclusion:The chief executive is, therefore, less well informed about problems at lower levels than are his or her subordinates at those levels.

没有越级禀报~levels没有缺失
(A) problems should be solved at the level in the management hierarchy at which they occur
--irrelevant
(B) employees should be rewarded for accurately reporting problems to their superiors
--weaken
(C) problem-solving ability is more important at higher levels than it is at lower levels of the management hierarchy
--irrelevant
(D) chief executives obtain information about problems at lower levels from no source other than their subordinates
--right
(E) some employees are more concerned about truth than about the way they are perceived by their superiors

--irrelevant
作者: leewonting    时间: 2012-4-12 17:33
78
背景:Most antidepressant drugs cause weight gain.
条件:Dieting can help reduce the amount of weight gained while taking such antidepressants. But some weight gain is unlikely to be preventable.
结论:SUPPORT
推测:选项应该给出一个例子,摄取这种药物的同时,即使很注意diet,也会胖
选C,(A,B看起来都挺对的- -)


79
背景:The fact is that 90% of the people in this country now report that they know someone who is unemployed
条件:A normal level of unemployment is 5%
结论:how about the Sharon’s argument ?
推测:两个人讨论的出发点不一样
选A


80
背景:A certain mayor has proposed a fee of 5 dollars per day on private vehicles entering the city, claiming that the fee will alleviate the city’s traffic congestion.
条件:The fee is higher than the round-trip bus fare
结论:The traffic situation will be better.
推测:车进入的费用比坐bus的费用还贵。那如果驾驶人宁愿花多点钱都要开车而不坐公交车呢?
选B


81
背景:No employee wants to be associated with bad news in the eyes of a superior.
条件:Information about serious problems at lower level is progressively softened and distorted as it goes up each step in the management hierarchy.
结论:Therefore, the chief executive is less well informed about problems
推测:选项如何题目所说就ok,上面的老板比下面的校老板知道坏消息的机会少。
选D,chief executive只能从他们的subordinate得到消息的话,那么就符合题目。
作者: Rena张    时间: 2012-4-20 11:01
1.
Time: 21''
Premise: antidepressant can cause weight gain. Dieting can help reduce the amount of weight gained while taking such antidepressants.
CONCLUSION: some weight gain is unlikely to be preventable.
Type: support which one of the following?
Choice: A C
2. 48''  A
3.      A B
4. 1'01''  B
5. 25''  D
精练题做错啦~~~
作者: sherryli    时间: 2012-5-9 15:42
Fact 1:  most antidepressant drugs cause weight gain, evern dieting can reduce the amount, some weight gain is inevitable    Ask for conclusion? What can we know from the fact?
A.    We cannot get this from the fact. The fact mentioned most antidepressant not all ; and dieting can reduce the amount of gain weight.
B.    Same as A.
C.    Correct
D.    The fact did not say this.
E.    The fact did not say this.
作者: FB小贝    时间: 2012-5-9 15:56
2. Most antidepressant drugs cause weight gain. While
dieting can help reduce the amount of weight gained
while taking such antidepressants, some weight gain
is unlikely to be preventable.
The information above most strongly supports which
one of the following?
Premise: drugs cause weight gain
Dieting can reduce the weight gain
Conclusion:  dieting can not reduce the weight gain while taking such drugs sometimes
Support: some weight gain is irrelevant to diet so that diet won’t help

(A) A physician should not prescribe any
antidepressant drug for a patient if that
patient is overweight.
Irrelevant

(B) People who are trying to lose weight should not
ask their doctors for an antidepressant drug.
irrelevant
(C) At least some patients taking antidepressant
drugs gain weight as a result of taking them.
so diet won’t necessarily help lose weight
correct
(D) The weight gain experienced by patients taking
antidepressant drugs should be attributed to
lack of dieting.
so diet helps, weaken
(E) All patients taking antidepressant drugs should
diet to maintain their weight.
irrelevant
作者: dwindwin1106    时间: 2012-5-16 03:26
(1)F:most a drugs cause weight gain.
F:while dieting can help reduce the amount of weight gained when taking such a drugs, some weight gain is unlikely to be preventable.
       Must be true
       Answer: C
       A.Irrelevant ------ 文章说most a drugs,而不是全部,因此该选项太绝对
       B.Irrelevant ------ 理由同A
       C.Correct ------ a drugs能引起增重,节食只能减少由药物引起的增重,但是增重并没停止,仍有一部分增重是不可避免的,所以至少有一些人会因a drugs而增重
       D.Irrelevant ------ 因果颠倒了。是a drugs本身引起了增重,并不是不节食引起了增重,而节食只是帮助减少drugs增加的重量
       E.Irrelevant ------ 绝对化


(2)R:90% of the people in this country report that they know someone who is unemployed.
S:a normal level of unemployment is 5%, with 1 out of 20 workers unemployed. At any given time, if a person knows 50 workers, 1 or more will very likely be unemployed.
Prephrase: unemployment并没有90%那么多
Answer: A
A.Correct
B.Irrelevant
C.Irrelevant
D.Irrelevant
E.Irrelevant


(3)(3) R:90% of the people in this country report that they know someone who is unemployed.
S:a normal level of unemployment is 5%, with 1 out of 20 workers unemployed. At any given time, if a person knows 50 workers, 1 or more will very likely be unemployed.
Assumption: S所指的地区并不是人口稀少的地区
Answer: B
A.Irrelevant
B.Correct
C.Irrelevant
D.Irrelevant
E.Irrelevant


(4)P:propose a fee of 5 dollars/day on private vehicles entering the city, claiming that the fee will alleviate the city’s traffic congestion.
P:the fee will exceed the cost of round-trip bus fare from many nearby points
C:many people will switch from using their cars to using the bus.
Weaken: 收费不能阻止private cars进入,如bus有些地方去不了?
Answer: EB

A.Irrelevant
B.Irrelevant correct------说明钱并不是问题所在,现在已经贵了但是还是有交通拥堵,那再征收费用也无济于事
C.irrelevant
D.Irrelevant
E.Correctirrelevant------还是有相当一部分private vehicles out of city造成了拥堵,那说明mayorproposal还是有效的


(5)P:because no employee wants to be bad in the eyes of a superior, information about serious problems at lower level is progressively softened and distorted as it goes up each step in the management hierarchy.
C:the chief executive is less well informed about problems at lower levels than are the subordinates at those levels
Assumption: information只能一层层向上汇报直到chief executive
Answer
A.Irrelevant
B.Irrelevant
C.Irrelevant
D.Correct
E.Irrelevant
作者: spencerX    时间: 2012-5-23 23:28
【精练】

B: Most antidepressant drugs cause weight gain. Diet can reduce the amount of weight gained.
Premise: Take such antidepressant drugs.
Conclusion:Some weight gain is unpreventable.
Pre: ?
Analysis:
A.    Irrelevant
B.    While “most antidepressant drugs” can cause weight gain, but it doesn’t mean that “all antidepressant drugs” will have the same effect. So we cannot conclude from the passage that this statement is ture.
C.    Correct
D.    “Most” antidepressant drugs can cause weight, so at least some antidepressant drugs will not cause weight gain.
E.    Out of scope

【逻辑链】

57.
Premise: moderate level of unemployment --- 5%
Conclusion:a person knows approximately 50 workers→1 or more will very likely be unemployed
Pre: The unemployment rate is not very high because people may know the same people who are unemployed
My answer:A?

58. My answer:B?

59.
B: mayor— $5- private vehicles entering the city- alleviate the city’s traffic congestion.
Premise: the fee will exceed the cost of round-trip bus fare from many nearby points
Conclusion: many people will switch from using their cars to using the bus
Pre: People would rather to endure the congestion.
My Answer: D

选项D中说的是commuter,但是mayor说的是people. 讨论范围不同
选项B:如果费用已经那么高了,但是人们还是选择用自己的car,那么说明再涨点价也不一定能够让他们用bus

60.
Premise: people don’t want to be associated with bad news in the eyes of a superior, so the information about problems will be hided.
Conclusion: executive knows less about problems than his or her subordinates.
Pre: There in no other ways for executive to know the problems.
My answer: D
作者: baseboss    时间: 2012-5-28 12:54
1.Take the drugs will lead to weight gain,While dieting can reduce the amount of weight gain, some weight gain is unlikely to be preventable.
Presume:
Anyway,take the drugs will lead to weight gain.

(A) A physician should not prescribe any
antidepressant drug for a patient if that
patient is overweight.
-->irrelevant
(B) People who are trying to lose weight should not
ask their doctors for an antidepressant drug.
-->irrelevant
(C) At least some patients taking antidepressant
drugs gain weight as a result of taking them.
-->correct
(D) The weight gain experienced by patients taking
antidepressant drugs should be attributed to
lack of dieting.
-->irrelevant
(E) All patients taking antidepressant drugs should
diet to maintain their weight.
-->irrelevant

2 5% unemployment level,1 out of 20 workers unemployment.If a person knows 50 workers,1 or more unemployment.
presume:
90% people know who unemployed is wrong.
D   A

3 5% unemployment level,1 out of 20 workers unemployment.If a person knows 50 workers,1 or more unemployment
presume:
base on the date distribute well
B

4 Fee is exceeded bus fare,people will choose bus rather than driving by car.
presume:
Take more time to take bus than car.People driving cost exceeded bus + fee.
B

5
presume:
1.report step by step,no skip-level report.
D
作者: 二楼往下掉    时间: 2012-6-11 00:05
1.
(1)20s support
(2)逻辑链
Background:
Most antidepressant drugs cause weight gain
Conclusion:
While dieting can help reduce the amount of weight gained while taking such antidepressants, some weight gain is unlikely to be preventable.
(3)有很多人进行了diet但是没有lose weight的效果
(4)A:已然超重就不要开这个药了,因为有的人即使节食也不好使,但是选项中提到了any,但文中是most,loser
B:这个药会增肥,所以减肥的人不要吃这个药,contender
C:大家吃这个药都会长胖,但是有的人可以通过节食减肥,有的人不可以,而不是有的人会长胖,shell game
D:有的人diet也减不下来,所以不是缺少diet的问题,loser
E:diet不是对谁都管用,loser
所以,选B
Must be true的问题一定是充分条件,但不一定是必要条件
正确答案C,我居然干掉了A没有干掉B。。。

2.
(1)35s draw conclusion
(2)逻辑链
Facts:
<1>90% of people in the country know someone is unemployed.
<2>moderate level unemployment is 5%——其实失业率不高

3.正确答案:B
我的答案:C

4.
(1)35s weaken
(2)逻辑链
Plan:
A fee of five dollars per day on private vehicles entering the city
Goal:
Alleviate the city's traffic congestion
Reason:
The fee will exceed the cost of round-trip bus fare from many nearby points, many people will switch from using their cars to using the bus
(3)这些人可能不会转移到bus,然后骑自行车,导致交通更拥堵
(4)A:这会使得更多人选择不开车,support,loser
B:说明这收钱的政策没必要,contender
C:这政策已然不是面向本来就坐车得人,所以这些人不在讨论范围,loser
D:人们反对但是并未指出政策的问题在哪里,loser
E:这些工作日开车的人并不是造成堵车的主要因素,所以这政策本身并不能很好的缓解交通堵塞,contender
在B中虽然已经有其他政策但是不妨碍新政策也会有很好的治理堵塞的效果,所以E靠谱
正确答案:B
这B和E怎么判断,我犹豫了好半天。。。结果还是错了。。。

5.
(1)45s assumption
(2)逻辑链
Primes:
Because no employee wants to be associated with bad news in the eyes of a superior, information about serious problems at lower levels is progressively softened and distorted as it goes up each step in the management hierarchy.
Conclusion:
The chief executive is less well informed about problems at lower levels than are his or her subordinates at those levels.
(3)假设领导们得到信息都是通过一层一层传达上来的
作者: jetyxo    时间: 2012-6-19 18:47
1  22"  background:antidepressant drugs lead to weight gaim
           premise:dieting reduce the amount of weight gian, but some weight gain is un-preventable
          my answer: B    people who ...
          right answer: C  at least some people...
+++++++++++++++++++++++++++++++++++=
2  20" R: 90% people in the country kuow somebody who is unemployment
          S: the moderate unemloyment rate is 5%,it means if a people know baout 50 person, one or more will likely be unemploement
          conclusion from S: the unemployment rate of this country is not very high--->A
------------------------------------------------------------------
3    S' arguement rely on the assumption that:  B
--------------------------------------------------------------------
4 23"  proposal: in order to alleviate the traffic congestion, goverment should collect 5 $ per car which enters the city
         reason: 5 $ is more than the cost of taking buses
         weaken: people who use the car doesn't care about the fee / taking bus has other expenditure
        A  support, C D  E irrelevant    answer:B
-----------------------------------------------------------------------
5 23"   Premise: serious problems become sofe when they go up
          conclusion: the top executive know less serious problem than their subordinate
          assumption: there is no way except from the subordinate for executive to know more about problems of low levels
          answer
作者: jiajiajudy    时间: 2012-7-2 05:34
1.    remise: drugs? weight gain
Premise: dieting can help reduce the amount of weight
Some weight gain is unlikely to be preventable
irrelevant
(B) irrelevant
(C) irrelevant
(D) irrelevant
(E) All patients taking antidepressant drugs should diet to maintain their weight.
2.    R:90% know someone who is unemployed
S: unemployment is 5%
The fact that 90% of the people know someone who is unemployed is not an indication that unemployment is abnormally high.
(B) irrelevant
(C) irrelevant
(D) irrelevant
(E) irrelevant
3.
irrelevant
(B) unemployment is not normally concentrated in geographically isolated segments of the population
(C) irrelevant
(D) irrelevant
(E) irrelevant
4.BG: a fee on private vehicles entering the city?alleviate traffic congestion
Premise fee exceed the cost of round-trip bus, many people will turn to bus
Not strong enough
(B) The cost of parking fees already makes it considerably more expensive for most people to take a private vehicle into the city than to take a bus.
(C) irrelevant
(D) weaken
(E) irrelevant
5. Premise: no employee wants to be associated with bad news? information  at lower levels is softened
Conclusion: chief less informed at lower levels than his subordinates at those levels
(A) irrelevant
(B) irrelevant
(C) irrelevant
(D) chief executives obtain information about problems at lower levels from no source other than their subordinates
(E) irrelevant
作者: cleotina    时间: 2012-7-7 09:54
7-7
35''

to support
1 a drug lead to weight gain
2 dieting will control the weight
c while talk about the drug, weight gain is inevitable
guess: take drug should also pay attention the weight gain issue, and dieting is one of the methods to solve the problems
OA: C
AB: irrelevant
D: the two terms above are not the causle relationship, so term one cannot infer term two
E: tricky, since the author not says that the dieting is the only way to solve the over weighting problem

54''
S lead to which conclusion
R: the alarming fact that more than 90% percent in the country knoew someone who is unemployed

S: moderate level of the unemployed is 5%, with i out of 20 worlers unemployed. that means each perspon know more than 50 person, one of them definitly is unemployed

guess: all the person who are unemployed are spread averagely
OA: A
others: can not infer

14''
assumption
the same as the qustion above
OA: B
ACD: can not infer
E: out of scope

35''
weaken
1: mayor propose to charge 5$/day for each bus enter the city
2: this mathod will elavate the traffic jam
3: this fee is higher than the bus ticket
guess: bus is not convient
      car can take more than one person, and the total bus fee of them are higher than 5$
      other terms that can prohibit this proposal
OA: B this show that people are not sensetive to the over cost
D: tricky, however, it not concern the term about the over cost, so it is not the best answer
others: irrelevant

25''
assumption
1 officers want to live a good impression to his boss, so he tend to give the good news.
2 if a bad news happen, they tend to solft and disorder it
C: so, the chief less know about the problem than a lower levle manager

guess: the author assume all the information that the chief got is from his direct subordinators

OA: D
AE: out of scope
BC: irrelevant
作者: LuckyYolandaLi    时间: 2012-7-7 15:46
1.
1
)计时:40s
2)逻辑链:
Background information: Most antidepressant drugs cause weight gain.
Premise: While dieting can help reduce the amount of weight gained while taking such antidepressants, some weight gain is unlikely to be preventable.
3)推测:Most antidepressant drugs cause some unpreventable weight gain.
4)选项分析:选C
A: Not mentioned.
B: Not mentioned.这个还有点像。。。
C: Correct.
D: Incorrect.
E: Incorrect.


2.
1
)计时:38s
2)逻辑链:
Background information: a normal, moderate level of unemployment is 5 percent, with 1 out of 20 workers unemployed.
Premise: So at any given time if a person knows approximately 50 workers, 1 or more will very likely be unemployed.
Conclusion:
3)推测:It is normal that 90 percent of the people in this country now report that they know someone who is unemployed if they know approximately 50 workers.
4)选项分析:选A
A: Correct.
B: Can’t be inferred.
C: Irrelevant.
D: Can’t be inferred. 这个选项句子结构我怎么没看明白。。。
E: Can’t be inferred


3.
1
)计时:s
2)逻辑链:
3)推测:Currently, unemployment is at a normal, moderate level.
4)选项分析:选B
刚开始想选A。。。


4.
1
)计时:35s
2)逻辑链:
Background information: A certain mayor has proposed a fee of five dollars per day on private vehicles entering the city, claiming that the fee will alleviate the city's traffic congestion.
Premise: the fee will exceed the cost of round-trip bus fare from many nearby points.
Conclusion: many people will switch from using their cars to using the bus.
3)推测:Most people who chose private vehicles don’t care about the cost so much.
4)选项分析:选E(正确答案:B
A: Irrelevant.
B: 刚开始觉得这个有点对。。。
C: Irrelevant.
D: 不选
E: Correct.


5.
1
)计时:42s
2)逻辑链:
Background information:
Premise: No employee wants to be associated with bad news in the eyes of a superior.
Intermediate: Information about serious problems at lower levels is progressively softened and distorted as it goes up each step in the management hierarchy.
Conclusion: The chief executive is less well informed about problems at lower levels than are his or her subordinates at those levels.
3)推测:Information about serious problems are reported upward step by step.
4)选项分析:选D
A: Irrelevant.
B: Counteractive.
C: Irrelevant.
D: Correct.
E: Counteractive.
作者: angelafeng    时间: 2012-7-11 18:22
精练
2. 19s
background information
Most antidepressant drugs cause weight gain.

Premise: While dieting can help reduce the amount of weight gained .

Conclusion:while taking such antidepressants, some weight gain is unlikely to be preventable.

Prephrase: (conclude)

Answer:C
(A) A physician should not prescribe any
antidepressant drug for a patient if that
patient is overweight.---------the “any” is too absolute, the background information of the argument just say “most”.
(B) People who are trying to lose weight should not
ask their doctors for an antidepressant drug.----------------people can dieting.
(C) At least some patients taking antidepressant
drugs gain weight as a result of taking them.---------------correct.

(D) The weight gain experienced by patients taking
antidepressant drugs should be attributed to
lack of dieting.--------------people taking the weight gain-causing antidepressants will gain weight regardless of whether they diet.

(E) All patients taking antidepressant drugs should
diet to maintain their weight.----------too absolute

PREP 练习
57.    (34655-!-item-!-188;#058&007580)26S
Roland:  The alarming fact is that 90 percent of the people in this country now report that they know someone who is unemployed.
Sharon:  But a normal, moderate level of unemployment is 5 percent, with 1 out of 20 workers unemployed.  So at any given time if a person knows approximately 50 workers, 1 or more will very likely be unemployed.
Sharon's argument is structured to lead to which of the following as a conclusion?
answer:A
(A) The fact that 90% of the people know someone who is unemployed is not an indication that unemployment is abnormally high.

(B) The current level of unemployment is not moderate.
(C) If at least 5% of workers are unemployed, the result of questioning a representative group of people cannot be the percentage Roland cites.
(D) It is unlikely that the people whose statements Roland cites are giving accurate reports.
(E) If an unemployment figure is given as a certain percent, the actual percentage of those without jobs is even higher.

58.    (34705-!-item-!-188;#058&007581)15s

Roland:  The alarming fact is that 90 percent of the people in this country now report that they know someone who is unemployed.
Sharon:  But a normal, moderate level of unemployment is 5 percent, with 1 out of 20 workers unemployed.  So at any given time if a person knows approximately 50 workers, 1 or more will very likely be unemployed.
Sharon's argument relies on the assumption that
answer:B

(A) normal levels of unemployment are rarely exceeded
(B) unemployment is not normally concentrated in geographically isolated segments of the population
(C) the number of people who each know someone who is unemployed is always higher than 90% of the population
(D) Roland is not consciously distorting the statistics he presents
(E) knowledge that a personal acquaintance is unemployed generates more fear of losing one's job than does knowledge of unemployment statistics

59.    (34753-!-item-!-188;#058&007583)27s
A certain mayor has proposed a fee of five dollars per day on private vehicles entering the city, claiming that the fee will alleviate the city's traffic congestion.  The mayor reasons that, since the fee will exceed the cost of round-trip bus fare from many nearby points, many people will switch from using their cars to using the bus.
Which of the following statements, if true, provides the best evidence that the mayor's reasoning is flawed?
answer:B
(A) Projected increases in the price of gasoline will increase the cost of taking a private vehicle into the city.
(B) The cost of parking fees already makes it considerably more expensive for most people to take a private vehicle into the city than to take a bus.
(C) Most of the people currently riding the bus do not own private vehicles.
(D) Many commuters opposing the mayor's plan have indicated that they would rather endure traffic congestion than pay a five-dollar-per day fee.
(E) During the average workday, private vehicles owned and operated by people living within the city account for twenty percent of the city's traffic congestion.

60.    (34801-!-item-!-188;#058&007584)31s
Because no employee wants to be associated with bad news in the eyes of a superior, information about serious problems at lower levels is progressively softened and distorted as it goes up each step in the management hierarchy.  The chief executive is, therefore, less well informed about problems at lower levels than are his or her subordinates at those levels.
The conclusion drawn above is based on the assumption that

Answer:D
(A) problems should be solved at the level in the management hierarchy at which they occur
(B) employees should be rewarded for accurately reporting problems to their superiors
(C) problem-solving ability is more important at higher levels than it is at lower levels of the management hierarchy
(D) chief executives obtain information about problems at lower levels from no source other than their subordinates
(E) some employees are more concerned about truth than about the way they are perceived by their superiors



作者: Donts    时间: 2012-7-16 12:39
1.计时30''premise:although dieting can help reduce the weight during the time taking the drug.conclusion:antidepressant drug can still cause weight gain.
推测:
(A) A physician should not prescribe any
antidepressant drug for a patient if that
patient is overweight. overweight和willingness to lose weight还是有别的
(B) People who are trying to lose weight should not
ask their doctors for an antidepressant drug.没有说明是不是因为药物导致weight gain.
(C) At least some patients taking antidepressant
drugs gain weight as a result of taking them.
(D) The weight gain experienced by patients taking
antidepressant drugs should be attributed to
lack of dieting.与drug无关。
(E) All patients taking antidepressant drugs should
diet to maintain their weight.重心在diet不在drug。
选C

虽然做对了,但是结构悲催地错了。。。
Premise: Most antidepressant drugs cause weight gain.
Premise: Dieting can help reduce the amount of weight gained while taking such antidepressants
Conclusion: Some weight gain is unlikely to be preventable.
作者: TICKCOCK    时间: 2012-8-15 22:13
逻辑小分队  8月15日   【1-15】
1.    C 看清加强题和归纳题的问题
2.    A
3.    B
4.    A (B)
5.    D
作者: wanggang0411    时间: 2012-8-23 18:02
1 21
Premise: Drugs cause weight gained.
Premise: On diet cause preventing part weight gained by drugs.
Must be true question.

2 31
R: The alarming fact is that 90 percent of people know someone who is unemployed.
S: The moderate level of unemployment is 5%.
Must be true question about S's conclusion:
I have no idea on this question.

3 20
Assumption question: The unemployement is not concentrated in geographically.

4 32
Premise: propose a fee.
Premise: The fee exceed the fare the cost of bus --> less private cars
Conclusion: This proposal will resolve the city congesstion.
Weakean:
by any reason, people cann't switch from car to bus.
E: the car is already in the city.

5 25
Premise: nobody want to esclate bad news ot their superior.
Premise: So the bad news become softly and distorted.
Conclusion: Chief executive knows less about these news than subordinates at those levels.
Assumption: There is no other way for Chief excuteive to get information.
D:p
作者: wanggang0411    时间: 2012-8-23 22:54
OK
Question 4:
This quesion is flaws in the reasoning, so the key point to resolve this question is to figure out what the mayor's reasoning is.
The answer is "If the cost of taking vehicle is higher than the cost of taking bus, commuters will swith to bus."
Obviously, answer B provide a evidence that the existed same cause didn't result the expected effect to contradict/refute/debate/dispute this reasoning.
作者: chengzaaaa    时间: 2012-9-1 08:49
901[1-15]
http://forum.chasedream.com/GMAT_CR/thread-619319-1-1.html
2.
1)17”aconclusion
2)antidepressant drugs?weight gain
Diet?reduce the weight gain
But some gain is not preventable.
3)antidepressant drugs?weight gain
4)B C
5)A. didn’t say anything about overweight, just about gaining weight
B. Correct. Taking antidepressant drugs is not going to help them with the goal.
C. ?
D. dieting could reduce the weight gain
E. it is not a problem of should or shouldn’t. it is that diet could reduce weight gain.

57.
1)27”conclusion
2)R: 90% of people say that they know someone who is unemployed.
S: normal: 5% unemployment?1 out of 20
?people know 50 workers?at normal rate?one or more likely to be unemployed
3)the fact that R said is not as alarming as it sounds
A

58.
1)10”assumption
2) R: 90% of people say that they know someone who is unemployed.
S: normal: 5% unemployment?1 out of 20
?people know 50 workers?at normal rate?one or more likely to be unemployed
3)…?
B

59.
1)17”weaken
2)5 dollars>the round-trip bus fare?people switch to bus, not cars
?
propose: charge 5 dollars a day for every private car entering the city?alleviate the city traffic
3)without 5 dollar fees, people are already paying more than 5 dollars for parking
B

60.
1)21”assumption
2)no worker wants to be linked with bad news in the boss’s eye?problem sound less serious as it goes up to top bosses
?CEO is less well informed than people at lower levels
3)CEO can’t learn about the problems through paths other than subordinates
D


谢谢lz!!!
作者: 我心匪席    时间: 2012-9-8 12:21
Time: 15’
Background Information: Antidepressant drugs cause weight gain.
Premises: dieting can help reduce the amount of weight gain.
Conclusion: Some weight gain is unlikely to be preventable.
Prephrase: Antidepressant will not cause diet disorder.
A-----irrelevant
B-----irrelevant
C-----irrelevant(right support)
D-----irrelevant
E-----right support

每次看题目一味追求速度,结果问题没看清,悲剧.....
================================================
Time: 24’
Background Information: 90% people report that they know someone who is unemployed.
Premises: Normal unemployment is 5 percent.
Conclusion: ?
A-----right
B-----irrelevant
C-----irrelevant
D-----irrelevant
E-----irrelevant
===========================================
Time: 24’
Background Information: 90% people report that they know someone who is unemployed.
Premises: Normal unemployment is 5 percent.
Conclusion: ?
A-----irrelevant
B-----right assumption
C-----irrelevant
D-----irrelevant
E-----irrelevant
==================================================
Time: 26’
Background Information: A fee of five dollar per day on private vehicles entering the city will alleviate the city’s traffic congestion.
Premises: The fee will exceed the cost of round-trip bus fare.
Conclusion: many people will prefer bus to cars.
Prephrase: Most traffic congestion comes from cars inner city.
A-----irrelevant
B-----irrelevant
C-----irrelevant
D-----irrelevant (right weaken)
E-----right weaken
百分比不能说明问题!
===============================================
Time: 24’
Premises: The problems at lower levels is softened and distorted as it goes up in the management hierarchy.
Conclusion: The chief is less well informed about problems at lower levels than are his or her subordinates at those levels.
A-----irrelevant
B-----irrelevant
C-----irrelevant
D-----right
E-----irrelevant
作者: ElenW    时间: 2012-10-15 10:07
精练(21'4)
pre1:Most antidepressant drugs cause weight gain
pre2:dieting can help reduce the amount of weight gained
con: taking such antidepressants, some weight gain is unlikely to be preventable.
(support)
PRE:the A and D are really different
Ans:B--Exaggerated
A-right??
C-not the issue---is right---attention:MOST
D-wrong
E-too strong--- Not all patients
逻辑
57(33'4)
Bg:90 percent of the people in this country now report that they know someone who is unemployed.

pre:moderate level of unempolyment is 5 percent-->1/20
pre:at any given time ,among 50 persons,1 or more will very like be unemployed.
(MAIN POINT)
Ans:A(凭的感觉............)留一个 看看大家的解析~~~
59(30'1)
Bg:A certain mayor has proposed a fee of five dollars per day on private vehicles entering the city
pre:the fee will exceed the cost of round-trip bus fare from many nearby points
pre:many people will switch from using their cars to using the bus.
con:the fee will alleviate the city's traffic congestion
(flaw of the reason)
Ans:B "already" indicate that the mayor's argument is likely to be attacked  .There is another reason to be the effect
作者: srafcatt    时间: 2012-10-20 08:59
精炼
strenghten 16s
antidepresent drugs gain weight
dieting cannot stop this.
my opinion:maybe this drug helps the nutrient absorbing.
choice:C
A irrelevant
B irrelevant
C correct
D the opposite of the fact in question stem
E irrelevant
---------------------------------------------------------------------
逻辑链
1.conclusion 21s
unemployment 5 percent,know 50 people one or more will be likely unemployed.
my conclusion: those unemployed people are well known
choice:A
A correct
B opposite of the fact
C irrelevant
D not true
E irrelevant
2.assumption 8s
same as the upwards
my assumption: this data is absolutely right
choice:A
A correct
B irrelevant details
C irrelevant
D not useful
E irrelevant
3.flaw 28s
5 dollars per private vehicles---traffic congestion
people tend to use bus for saving money
my opinion:may be those rich people just don't care about money
choice: D
A irrelevant
B irrelevant
C irrelevant
D correct
E irrelevant,or opposite of the fact
4.assumption 31s
employees tend to hide bad news--softened distorted---chief executive not well informed about the truth.
my assumption:the chief executive get information only from those reports form lower levels.
choice
A irrelevant
B irrelevant
C irrelevant
D correct
E not true
作者: srafcatt    时间: 2012-10-20 09:17
纠错:
Roland:  The alarming fact is that 90 percent of the people in this country now report that they know someone who is unemployed.

Sharon:  But a normal, moderate level of unemployment is 5 percent, with 1 out of 20 workers unemployed.  So at any given time if a person knows approximately 50 workers, 1 or more will very likely be unemployed.

Sharon's argument relies on the assumption that

(A) normal levels of unemployment are rarely exceeded
(B) unemployment is not normally concentrated in geographically isolated segments of the population
(C) the number of people who each know someone who is unemployed is always higher than 90% of the population
(D) Roland is not consciously distorting the statistics he presents
(E) knowledge that a personal acquaintance is unemployed generates more fear of losing one's job than does knowledge of unemployment statistics
90%的人都声称认识失业的朋友。
失业率5%的话,就是20人中就有1个人失业,因此你的朋友数量超过50的话,你就非常有可能认识一个失业的人啊。(因此90%的人说他们认识失业的人,这很正常)

为了解释第一句话的现象,第二句话给出了一个理论,其实挺有道理。只要是保证这个5%的失业人口平均的分配在整个国家中。但是如果这5%的人,都在一个地区(比如说都在上海市)。那么第二句话就不会成立的。
至于A 为什么错,文章中讲失业率5%,因此,是否经常超过这个数目和这一次是没有关系的。
59.    (34753-!-item-!-188;#058&007583)

A certain mayor has proposed a fee of five dollars per day on private vehicles entering the city, claiming that the fee will alleviate the city's traffic congestion.  The mayor reasons that, since the fee will exceed the cost of round-trip bus fare from many nearby points, many people will switch from using their cars to using the bus.

Which of the following statements, if true, provides the best evidence that the mayor's reasoning is flawed?

(A) Projected increases in the price of gasoline will increase the cost of taking a private vehicle into the city.
(B) The cost of parking fees already makes it considerably more expensive for most people to take a private vehicle into the city than to take a bus.
(C) Most of the people currently riding the bus do not own private vehicles.
(D) Many commuters opposing the mayor's plan have indicated that they would rather endure traffic congestion than pay a five-dollar-per day fee.
(E) During the average workday, private vehicles owned and operated by people living within the city account for twenty percent of the city's traffic congestion.
题目分析:A说汽油价格上升将会增加私有车进城的成本,没说过。B说停车费已经(记住是已经而不是将要)让开车比做bus贵了,明显用了以其人之道还治其人之身的驳斥方法,你原文不是说增加进城成本吗?现在我就在用这招,可交通还是不好,留着,A可说的是将要,那就没有用到这个驳斥方法。(如果各位绕不过来,算了,反正B是沾边的,先留着再说。)C说坐bus的人没有私家车,没说,杀。D很多反对者说他们宁愿交通堵一点也不愿意交钱,没说过原文有人有这个态度,杀。E说私家车占的比例问题,没说,杀。---来自

总结:有一种驳斥方法是:就算你的逻辑对(多花钱可以抑制开私家车),你不希望的结果还是有(停车费比做公交甚至加上你的5块钱多不知多少,还是开私家车)。
你对(逻辑对),但你错(结果错)。
缺陷题要在文中找缺陷,这和weaken或assumption是截然不同的


作者: zxppx    时间: 2013-1-15 10:53
2.

Most antidepressant drugs cause weight gain. While
dieting can help reduce the amount of weight gained
while taking such antidepressants, some weight gain
is unlikely to be preventable.

In order to strengthen the argument, we should point out that those antidepressant drugs can cause weight while with dieting. But please note the question, which refers to support one of the following answers, rather than use the answer to support the argument. I ignore the meaning of the argument!
The information above most strongly supports which
one of the following?

(A) A physician should not prescribe any
antidepressant drug for a patient if that
patient is overweight.

Overweight is outside the scope of the argument.
(B) People who are trying to lose weight should not
ask their doctors for an antidepressant drug.

People who are trying to lose weight have no effect on the argument.
(C) At least some patients taking antidepressant
drugs gain weight as a result of taking them.

It seems to be a contender, but it does not refer to diet.

Please note the limitative words in the argument, such as some, most, and such. So this answer can be concluded from the argument.
(D) The weight gain experienced by patients taking
antidepressant drugs should be attributed to
lack of dieting.

Even lack of dieting, some people would still gain weight. BA

In fact, this answer strengthens the argument, rather as a conclusion.
(E) All patients taking antidepressant drugs should
diet to maintain their weight.

Even though this could be true, it does not strengthen the argument.

The answer is too strong, because some antidepressant medication cannot cause weight gain.



57. (34655-!-item-!-188;#058&007580)
Roland: The alarming fact is that 90 percent of the people in this country now report that they know someone who is unemployed.
Sharon: But a normal, moderate level of unemployment is 5 percent, with 1 out of 20 workers unemployed. So at any given time if a person knows approximately 50 workers, 1 or more will very likely be unemployed.
Sharon's argument is structured to lead to which of the following as a conclusion?
(A) The fact that 90% of the people know someone who is unemployed is not an indication that unemployment is abnormally high.

More persons someone knows, more unemployment will be knew by the person. But this does not make sure that the unemployment is really high. BA
(B) The current level of unemployment is not moderate.

We know nothing about the current level of unemployment, even though we know the percentage of the people who know someone is unemployed. These two are different.
(C) If at least 5% of workers are unemployed, the result of questioning a representative group of people cannot be the percentage Roland cites.

Roland does not question the representation of a group of people.
(D) It is unlikely that the people whose statements Roland cites are giving accurate reports.

S does not question R’s argument, but rather tries to explain it.
(E) If an unemployment figure is given as a certain percent, the actual percentage of those without jobs is even higher.

Even S gives a moderate level of unemployment, but we do not the actual unemployment rate.



58. (34705-!-item-!-188;#058&007581)
Roland: The alarming fact is that 90 percent of the people in this country now report that they know someone who is unemployed.
Sharon: But a normal, moderate level of unemployment is 5 percent, with 1 out of 20 workers unemployed. So at any given time if a person knows approximately 50 workers, 1 or more will very likely be unemployed.
Sharon's argument relies on the assumption that

(A) normal levels of unemployment are rarely exceeded

We do not know whether the normal levels of unemployment are rarely exceeded or not.
(B) unemployment is not normally concentrated in geographically isolated segments of the population

I think it is the best answer. Because at the beginning of Sharon’s reasoning, we can find the ‘But’, which means that opinions different from Roland’s. BA

In assumption question, we can use the negation way to estimate the right answer.
(C) the number of people who each know someone who is unemployed is always higher than 90% of the population

We know nothing about this.
(D) Roland is not consciously distorting the statistics he presents

The same to C. It seems to be a contender.
(E) knowledge that a personal acquaintance is unemployed generates more fear of losing one's job than does knowledge of unemployment statistics

Actually, it is an assumption of argument of Roland.


59. (34753-!-item-!-188;#058&007583)
A certain mayor has proposed a fee of five dollars per day on private vehicles entering the city, claiming that the fee will alleviate the city's traffic congestion. The mayor reasons that, since the fee will exceed the cost of round-trip bus fare from many nearby points, many people will switch from using their cars to using the bus.
Which of the following statements, if true, provides the best evidence that the mayor's reasoning is flawed?
(A) Projected increases in the price of gasoline will increase the cost of taking a private vehicle into the city.

Actually, this answer choice points out an alternative cause to make the argument valid.
(B) The cost of parking fees already makes it considerably more expensive for most people to take a private vehicle into the city than to take a bus.

Since the argument reveals that the mayor reason of the validity of the plan is the fee will exceed the cost of round-trip bus fare from many nearby points, this answer choice point out the reason is fallacious.BA
(C) Most of the people currently riding the bus do not own private vehicles.

It does not weaken the argument, since these people cannot drive cars to the city.
(D) Many commuters opposing the mayor's plan have indicated that they would rather endure traffic congestion than pay a five-dollar-per day fee.

It seems like a contender, but a more thorough analysis reveals that it is not reasonable. Because these drivers have to endure both traffic congestion and a five-dollar-per day fee, the line of reasoning of this choice cannot stand.
(E) During the average workday, private vehicles owned and operated by people living within the city account for twenty percent of the city's traffic congestion.

The percentage of the private vehicles accounts for the city’s traffic congestion is irrelevant.



60. (34801-!-item-!-188;#058&007584)
Because no employee wants to be associated with bad news in the eyes of a superior, information about serious problems at lower levels is progressively softened and distorted as it goes up each step in the management hierarchy. The chief executive is, therefore, less well informed about problems at lower levels than are his or her subordinates at those levels.
The conclusion drawn above is based on the assumption that
(A) problems should be solved at the level in the management hierarchy at which they occur

Solving problems are not referred in the argument.
(B) employees should be rewarded for accurately reporting problems to their superiors

Reward is outside the scope of the argument.
(C) problem-solving ability is more important at higher levels than it is at lower levels of the management hierarchy

Problem-solving ability is irrelevant.
(D) chief executives obtain information about problems at lower levels from no source other than their subordinates

In this case, it accurately points out the gap that the source of information obtained by chief executives. BA
(E) some employees are more concerned about truth than about the way they are perceived by their superiors

If this is the case, the answer choice will weaken the argument.





作者: deniseya    时间: 2013-1-15 11:21
1. B(C pay attention to "most")
(A) A physician should not prescribe any
antidepressant drug for a patient if that
patient is overweight.--------------------may not gain much after diet
(B) People who are trying to lose weight should not
ask their doctors for an antidepressant drug.----------correct
(C) At least some patients taking antidepressant
drugs gain weight as a result of taking them.-------just support P
(D) The weight gain experienced by patients taking
antidepressant drugs should be attributed to
lack of dieting.----------Against P(unlikely to be preventable)
(E) All patients taking antidepressant drugs should
diet to maintain their weight.---------againstP(unlikely to be preventable)

2.1'18''
A
(A) The fact that 90% of the people know someone who is unemployed is not an

indication that unemployment is abnormally high.------correct
(B) The current level of unemployment is not moderate.-----irrelevent
(C) If at least 5% of workers are unemployed, the result of questioning a

representative group of people cannot be the percentage Roland cites.----against P
(D) It is unlikely that the people whose statements Roland cites are giving

accurate reports.----against P
(E) If an unemployment figure is given as a certain percent, the actual percentage

of those without jobs is even higher.-----irrelevent

3.1'
B
(A) normal levels of unemployment are rarely exceeded------irrelevent
(B) unemployment is not normally concentrated in geographically isolated segments

of the population----correct
(C) the number of people who each know someone who is unemployed is always higher

than 90% of the population------against P
(D) Roland is not consciously distorting the statistics he presents-----that's not

an assumption of S
(E) knowledge that a personal acquaintance is unemployed generates more fear of

losing one's job than does knowledge of unemployment statistics----irrelevent

4.1'24''
B
(A) Projected increases in the price of gasoline will increase the cost of taking a private vehicle into the city.----irrelevent
(B) The cost of parking fees already makes it considerably more expensive for most people to take a private vehicle into the city than to take a bus.---correct
(C) Most of the people currently riding the bus do not own private vehicles.----irrelevent
(D) Many commuters opposing the mayor's plan have indicated that they would rather endure traffic congestion than pay a five-dollar-per day fee.----irrelevent
(E) During the average workday, private vehicles owned and operated by people living within the city account for twenty percent of the city's traffic congestion.---that's a support


5.1'20''
D
(A) problems should be solved at the level in the management hierarchy at which they occur----irrelevent
(B) employees should be rewarded for accurately reporting problems to their superiors-----weaken
(C) problem-solving ability is more important at higher levels than it is at lower levels of the management hierarchy---irrelevent
(D) chief executives obtain information about problems at lower levels from no source other than their subordinates-----correct
(E) some employees are more concerned about truth than about the way they are perceived by their superiors ------weaken



作者: pennyz    时间: 2013-2-18 16:34
补作业
1:50 support
c
premise:while dieting can reduce the amount of weight
       but some weight is unlikely to prevent
conclusion: most antidepressant drugs cause weight gain

A) A physician should not prescribe any
antidepressant drug for a patient if that
patient is overweight.-------if a is true then b is true,any 太绝对
(B) People who are trying to lose weight should not
ask their doctors for an antidepressant drug.-----?
(C) At least some patients taking antidepressant
drugs gain weight as a result of taking them.---correct?
(D) The weight gain experienced by patients taking
antidepressant drugs should be attributed to
lack of dieting.-------weaken
(E) All patients taking antidepressant drugs should
diet to maintain their weight.------irrelevant
作者: pennyz    时间: 2013-2-18 16:49
1:08s
conclusion
a
roland:90% people know someone is unemployed, this fact is alarming
s:the moderate rate of umemployment is 5%
 if a person knows more than 50 peole ,he will know one unemployed person or more

s means the situation is not so alarming
1:36s
assumption
b
unclear
???
1:25s
weaken
b
premise: the cost of private car is much than the public commutor,and lead the commuter to take bus
conclusion:level 5 dollar for private vehicles can relieve the congestion
rephrase:the cost of private vehicles has already surplus the cost of public commutors
54s
assumption
d
premise: no one want their supremier know the bad result of work
conclusion:the chief executives know less than his subordinates
作者: CD用户825193    时间: 2013-2-24 19:49
标题: Daily CR-15_2013-02-25

" (Weak / Support)
P:
P:
C:
Pre:

Answers:


" (Weak / Support)
P:
P:
C:
Pre:

Answers:



" (Weak / Support)
P:
P:
C:
Pre:

Answers:



" (Weak / Support)
P:
P:
C:
Pre:

Answers:



" (Weak / Support)
P:
P:
C:
Pre:

Answers:


作者: Feelalive    时间: 2013-7-13 14:44
Feelalive到此一游
作者: Extremo    时间: 2013-7-21 18:42
mark~
作者: Elisha728    时间: 2013-9-1 11:44
CACBD
6'33''
作者: Elisha728    时间: 2013-9-1 11:53
58.    (34705-!-item-!-188;#058&007581)

Roland:  The alarming fact is that 90 percent of the people in this country now report that they know someone who is unemployed.
Sharon:  But a normal, moderate level of unemployment is 5 percent, with 1 out of 20 workers unemployed.  So at any given time if a person knows approximately 50 workers, 1 or more will very likely be unemployed.
Sharon's argument relies on the assumption that
(A) normal levels of unemployment are rarely exceeded
(B) unemployment is not normally concentrated in geographically isolated segments of the population
(C) the number of people who each know someone who is unemployed is always higher than 90% of the population
(D) Roland is not consciously distorting the statistics he presents
(E) knowledge that a personal acquaintance is unemployed generates more fear of losing one's job than does knowledge of unemployment statistics

For assumption question, we can use negation technique.
For option B, the sentence after negation should be:
Unemployment is normally concentrated in geographically isolated segments of the population.

If so, the data is not representative and hence the conclusion of Shannon’s argument that

“So at any given time if a person knows approximately 50 workers, 1 or more will very likely be unemployed” since the data is not representative enough, hence it can’t be used to prove that the country is not undergoing high unemployment rate.

Only the negation of B can make Shannon’s conclusion unwarranted, so B is the correct answer.

The trick of this question is to attack the credibility of supporting data





作者: lyrsilvia    时间: 2013-9-19 19:38
C ABBD
1. 17’
BG: Most antidepressant drugs cause weight gain
P: dieting can help reduce the amount of weight gained while taking such antidepressants
C: some weight gain is unlikely to be preventable
SUPPORT WHICH OF THE FOLLOWING: antidepressant drugs did cause  weight gain
答案:C
(A) A physician should not prescribe any antidepressant drug for a patient if that patient is overweight.-----------is there only one criterion for the prescribe of antidepressant drug? if the depressant level is too high and to help the patients with the more serious disease. most≠all
(B) People who are trying to lose weight should not ask their doctors for an antidepressant drug.-----------just as the reason for answer A, if people have bad depress, then whether they are losing weight is not the case.most≠all
(C) At least some patients taking antidepressant drugs gain weight as a result of taking them.-------CORRECT. 取非no patients gain weight推翻整个premise的Information.
(D) The weight gain experienced by patients taking antidepressant drugs should be attributed to lack of dieting.--------------weaken. the premise states the weight gain is due to the antidepressant drugs.
(E) All patients taking antidepressant drugs should diet to maintain their weight.--------what patients should do is not supported by the information

2. 19'
R: 90% of people in this country know someone who is unemployed
S: moderate level of unemployment is 5%. if a person knows 50 worker, 1 or more will very likely be unemployed
CONCLUSION:  the unemployment rate is not high
答案:A
(A) The fact that 90% of the people know someone who is unemployed is not an indication that unemployment is abnormally high.----------------------CORRECT. the more workers people know, the more likely to know someone is unemployed
(B) The current level of unemployment is not moderate.-----------it is just R's imply
(C) If at least 5% of workers are unemployed, the result of questioning a representative group of people cannot be the percentage Roland cites.---------------means R city is not the usual one. support R but not conclude from S
(D) It is unlikely that the people whose statements Roland cites are giving accurate reports.----------whether the reports is accurate or not has no evidence to prove.
(E) If an unemployment figure is given as a certain percent, the actual percentage of those without jobs is even higher.----------not what S want to mean. be careful about the BUT in S's claim

3. 19'
R: 90% of people in this country know someone who is unemployed
S: moderate level of unemployment is 5%. if a person knows 50 worker, 1 or more will very likely be unemployed
ASSUMPTION: one people can knows more than 50 workers
答案:B
(A) normal levels of unemployment are rarely exceeded--------------if always exceeded, what conclusion? 在这里有犹豫,gmatclub上面说If normal levels of unemployment were exceeded relatively frequently, and if Roland's figure of 90 percent were an exaggeration, Sharon's argument would be unaffected, so choices A and D are incorrect. 相比B项还是不够完整。  
(B) unemployment is not normally concentrated in geographically isolated segments of the population---------------CORRECT. if is concentrated, people will not have to know 50 workers to get 1 or more unemployed worker.
(C) the number of people who each know someone who is unemployed is always higher than 90% of the population-----------------if not, no conclusion can be drawn
(D) Roland is not consciously distorting the statistics he presents------------if consciously, no conclusion can be drawn
(E) knowledge that a personal acquaintance is unemployed generates more fear of losing one's job than does knowledge of unemployment statistics---------------what about fear is not relevant.

4. 18'
P: a fee of five dollars per day on private vehicles entering the city, the fee will exceed the cost of round-trip bus fare from many nearby points
C: many people will switch from using their cars to using the bus; the fee will alleviate the city's traffic congestion
REASONING FLAW: fee≠fewer car
答案:B
(A) Projected increases in the price of gasoline will increase the cost of taking a private vehicle into the city.---------------gasoline increase is another project. irrelevant
(B) The cost of parking fees already makes it considerably more expensive for most people to take a private vehicle into the city than to take a bus.-----------------CORRECT. means the current higher cost does not reduce the car entering the city, maybe higher cost cannot generate fewer cars.
(C) Most of the people currently riding the bus do not own private vehicles.--------------whether people own private vehicles is not relevant
(D) Many commuters opposing the mayor's plan have indicated that they would rather endure traffic congestion than pay a five-dollar-per day fee.-----------------strengthen the reasoning. not flaw
(E) During the average workday, private vehicles owned and operated by people living within the city account for twenty percent of the city's traffic congestion.--------------what the percentage of private vehicles is irrelevant for justifying the mayor's reasoning.

5. 14'
BG: no employee wants to be associated with bad news in the eyes of a superior
P: information about serious problems at lower levels is progressively softened and distorted as it goes up each step in the management hierarchy
C:  chief executive is less well informed about problems at lower levels than are his or her subordinates at those levels.
ASSUMPTION:  chief executive just has one way to know the situation?
答案:D
(A) problems should be solved at the level in the management hierarchy at which they occur-------when the problem should be solved is irrelevant.
(B) employees should be rewarded for accurately reporting problems to their superiors----------not assumption. this just a solution for the bad situation.
(C) problem-solving ability is more important at higher levels than it is at lower levels of the management hierarchy--------------------problem solving ability is not discussed
(D) chief executives obtain information about problems at lower levels from no source other than their subordinates-----------------CORRECT. if other source are available, then chief executive can be well informed.
(E) some employees are more concerned about truth than about the way they are perceived by their superiors --------------------- if concerned more about truth, then chief executive will not be less informed.
作者: Mint静默    时间: 2013-9-24 20:25
1) 计时:25s
2) 逻辑链
Background Information:
Most antidepressant drugs cause weight gain.
Premise:
While dieting can help reduce the amount of weight gained while taking such antidepressants, some weight gain is unlikely to be preventable.
3) 推测:(conclusion)
吃了antidepressant的药的人注定长膘
4) 选项分析:
(A) A physician should not prescribe any antidepressant drug for a patient if that patient is overweight. 题目中说most antidepressant drugs 会长膘,但是还是有一部分drugs不会长膘的
(B) People who are trying to lose weight should not ask their doctors for an antidepressant drug. 同A
(C) At least some patients taking antidepressant drugs gain weight as a result of taking them.
(D) The weight gain experienced by patients taking antidepressant drugs should be attributed to lack of dieting.
(E) All patients taking antidepressant drugs should diet to maintain their weight.

作者: 览物之情    时间: 2013-10-16 16:53
1 23s
2.
Premise: Most antidepressant drugs cause weight gain. While dieting can help reduce the amount of weight gained
while taking such antidepressants, some weight gainis unlikely to be preventable.
The information above most strongly supports which one of the following?
P Must be true
(A) A physician should not prescribe any antidepressant drug for a patient if that patient is overweight.---NM
(B) People who are trying to lose weight should not
ask their doctors for an antidepressant drug.-----might
(C) At least some patients taking antidepressant
drugs gain weight as a result of taking them.-----correct
(D) The weight gain experienced by patients taking
antidepressant drugs should be attributed to
lack of dieting.----------wrong
(E) All patients taking antidepressant drugs should
diet to maintain their weight.-----wrong
重点在区分might and correct

2 32s演绎推理
Roland:  The alarming fact is that 90 percent of the people in this country now report that they know someone who is unemployed.
Sharon:  But a normal, moderate level of unemployment is 5 percent, with 1 out of 20 workers unemployed.  So at any given time if a person knows approximately 50 workers, 1 or more will very likely be unemployed.


Sharon's argument is structured to lead to which of the following as a conclusion? Must be true 说明90%的人认识more than 20 people.百分比之间没有绝对的关系吧
(A)        The fact that 90% of the people know someone who is unemployed is not an indication that unemployment is abnormally high.---correct
(B) The current level of unemployment is not moderate.---NM
(C) If at least 5% of workers are unemployed, the result of questioning a representative group of people cannot be the percentage Roland cites.---wrong
(D) It is unlikely that the people whose statements Roland cites are giving accurate reports.-----wrong
(E) If an unemployment figure is given as a certain percent, the actual percentage of those without jobs is even higher.---NM



3 30s
Sharon: premise: But a normal, moderate level of unemployment is 5 percent, with 1 out of 20 workers unemployed. Conclusion: So at any given time if a person knows approximately 50 workers, 1 or more will very likely be unemployed.

Sharon's argument relies on the assumption that P
(A) normal levels of unemployment are rarely exceeded---NM
(B) unemployment is not normally concentrated in geographically isolated segments of the population----correct
(C) the number of people who each know someone who is unemployed is always higher than 90% of the population---NM
(D) Roland is not consciously distorting the statistics he presents---NM
(E) knowledge that a personal acquaintance is unemployed generates more fear of losing one's job than does knowledge of unemployment statistics--NM


4 24s
Plan:A certain mayor has proposed a fee of five dollars per day on private vehicles entering the city, claiming that the fee will alleviate 减轻the city's traffic congestion.  Premise:The mayor reasons that, since the fee will exceed the cost of round-trip bus fare from many nearby points, many people will switch from using their cars to using the bus.

Which of the following statements, if true, provides the best evidence that the mayor's reasoning is flawed?不是方案评估因为已经评价了方案是flawed.现在找原因。是因果推理的问题
方案评估 anyside effect or other factors need to be considered.
(A)        Projected increases in the price of gasoline will increase the cost of taking a private vehicle into the city.----和premise一样
(B) The cost of parking fees already makes it considerably more expensive for most people to take a private vehicle into the city than to take a bus.---- 说明因果没有联系,方案不能达到效果
(C) Most of the people currently riding the bus do not own private vehicles.—不能证明有车的做bus没用
(D) Many commuters opposing the mayor's plan have indicated that they would rather endure traffic congestion than pay a five-dollar-per day fee.---Irrelevant.目的是solve traffic congestion喜不喜欢不重要
(E) During the average workday, private vehicles owned and operated by people living within the city account for twenty percent of the city's traffic congestion.----只占20%不能说明不可以solve

4 19s
Premise:Because no employee wants to be associated with bad news in the eyes of a superior, information about serious problems at lower levels is progressively softened and distorted as it goes up each step in the management hierarchy.
Conclusion: The chief executive is, therefore, less well informed about problems at lower levels than are his or her subordinates at those levels.

The conclusion drawn above is based on the assumption that
因果推理排除干扰因素
(A)        problems should be solved at the level in the management hierarchy at which they occur---NM
(B) employees should be rewarded for accurately reporting problems to their superiors-----NM
(C) problem-solving ability is more important at higher levels than it is at lower levels of the management hierarchy-----NM
(D) chief executives obtain information about problems at lower levels from no source other than their subordinates----correct
(E) some employees are more concerned about truth than about the way they are perceived by their superiors ___wrong

作者: w.melhere    时间: 2013-10-28 11:43
今天状态好差啊,早上被乱七八糟的事忙昏了头,做的一点都不好。泪~

1. 20‘错了。逻辑理解有偏差。错选了B,但实际B 和A的错误类似,绝对的主观判断。
2. 31’ 这个做过
3. 没有做过这道题的假设,虽然对了,但不是很有把握。B应该是说失业人口并不集中在某一个分割开的区域,说明S的数据是可信的。
4.30‘ 又错了。面壁戳墙一百下。C选项读题时不认真,理解有误。B说现在的停车费已经很高,仍然很多人开车进入,说明价格并不能较大程度上影响是否开车进入城市。削弱了计划的有效性。
5.31’做过~
作者: yuehuasunday    时间: 2013-11-17 14:41
1-15

1.

Background: Most antidepressant drugs cause weight gain.

Premise: While
dieting can help reduce the amount of weight gained
while taking such antidepressants

Conclusion: some weight gain
is unlikely to be preventable.

©

2.

Background: Roland:  The alarming fact is that 90 percent of the people in this country now report that they know someone who is unemployed.

Premise: Sharon:  But a normal, moderate level of unemployment is 5 percent, with 1 out of 20 workers unemployed.  So at any given time if a person knows approximately 50 workers, 1 or more will very likely be unemployed.

Conclusion: Sharon's argument is structured to lead to which of the following as a conclusion?

(A) Correct

3.

Background: A certain mayor has proposed a fee of five dollars per day on private vehicles entering the city, claiming that the fee will alleviate the city's traffic congestion.

Premise: The mayor reasons that, since the fee will exceed the cost of round-trip bus fare from many nearby points.

Conclusion: Many people will switch from using their cars to using the bus.

(B) Correct

4.

Background: Because no employee wants to be associated with bad news in the eyes of a superior

Premise: information about serious problems at lower levels is progressively softened and distorted as it goes up each step in the management hierarchy.

Conclusion: therefore, less well informed about problems at lower levels than are his or her subordinates at those levels.

(D) Correct


作者: irenetopia    时间: 2013-11-28 15:49


精练题:1:10

Most antidepressant drugs cause weight gain. While
dieting can help reduce the amount of weight gained
while taking such antidepressants, some weight gain
is unlikely to be preventable.
The information above most strongly supports which
one of the following?

一些anti药会引起weight gain
Dieting可以帮助下降weight gain
但是有些weight gain是不会因此下滑的
问must true:结合信息,即大部分!!!anti药会使人肥胖,dieting有时可以降低这个肥胖!!!!但不是100%都能降!!!!

(A) A physician should not prescribe any
antidepressant drug for a patient if that
patient is overweight.——谁说哒!Generate了
(B) People who are trying to lose weight should not
ask their doctors for an antidepressant drug.——越吃越胖,跟题干相反了
(C) At least some patients taking antidepressant
drugs gain weight as a result of taking them.——bingo!Most和at least对应,很谨慎
(D) The weight gain experienced by patients taking
antidepressant drugs should be attributed to
lack of dieting.——错了,有的weight gain用了dieting也没用
(E) All patients taking antidepressant drugs should
diet to maintain their weight.——all就错了……题干中没有一次出现all这么绝对的词


PREP 练习

57.   36S
Roland:  The alarming fact is that 90 percent of the people in this country now report that they know someone who is unemployed.

Sharon:  But a normal, moderate level of unemployment is 5 percent, with 1 out of 20 workers unemployed.  So at any given time if a person knows approximately 50 workers, 1 or more will very likely be unemployed.

Sharon's argument is structured to lead to which of the following as a conclusion?
逻辑链R:这个国家90%报告说他们知道谁失业
S:通常失业率是在5%,20个人里面有一个失业,所以,如果一个人认识50个worker,里面至少有一个是失业的。
问S的结论:代表性问题,即认识的人越多,知道失业的人就越多,这个90%的人认识的人群不是互相独立的
就是可能90%的人中有人知道的失业的人是重复的
即其实没有那么多人失业

(A) The fact that 90% of the people know someone who is unemployed is not an indication that unemployment is abnormally high.——bingo!
(B) The current level of unemployment is not moderate.——无关
(C) If at least 5% of workers are unemployed, the result of questioning a representative group of people cannot be the percentage Roland cites.——因果倒置了
(D) It is unlikely that the people whose statements Roland cites are giving accurate reports.——无关
(E) If an unemployment figure is given as a certain percent, the actual percentage of those without jobs is even higher.——反了

58.   52S
Roland:  The alarming fact is that 90 percent of the people in this country now report that they know someone who is unemployed.

Sharon:  But a normal, moderate level of unemployment is 5 percent, with 1 out of 20 workers unemployed.  So at any given time if a person knows approximately 50 workers, 1 or more will very likely be unemployed.

Sharon's argument relies on the assumption that

逻辑链,跟上题一样
思路:即讨论样本的代表性,S认为失业率平均下来是5%是平均分布的
(A) normal levels of unemployment are rarely exceeded——错误,对S结论没有任何影响(B) unemployment is not normally concentrated in geographically isolated segments of the population——bingo!如果是集中在一个群体的话,就不成立了
(C) the number of people who each know someone who is unemployed is always higher than 90% of the population——无关
(D) Roland is not consciously distorting the statistics he presents——没提到R的代表性。是une的代表性
(E) knowledge that a personal acquaintance is unemployed generates more fear of losing one's job than does knowledge of unemployment statistics——knowledge。。。排除!

59.    1:13
A certain mayor has proposed a fee of five dollars per day on private vehicles entering the city, claiming that the fee will alleviate the city's traffic congestion.  The mayor reasons that, since the fee will exceed the cost of round-trip bus fare from many nearby points, many people will switch from using their cars to using the bus.

Which of the following statements, if true, provides the best evidence that the mayor's reasoning is flawed?
逻辑链:为了解决交通堵塞,市长决定向进城的私家车主收取5D每天的费用,因为这个费用比搭巴士贵,所以,可以缓解交通。
问削弱:思路:判定这个方案,从这几个方面找选项:1,真的有效吗?2,真的能达到吗?3,副作用4,方案本身问题

(A) Projected increases in the price of gasoline will increase the cost of taking a private vehicle into the city.——能不能影响到交通堵塞?无关~pass
(B) The cost of parking fees already makes it considerably more expensive for most people to take a private vehicle into the city than to take a bus.——bingo!典型同因异果。
(C) Most of the people currently riding the bus do not own private vehicles. ——能不能影响到交通堵塞?无关~pass
(D) Many commuters opposing the mayor's plan have indicated that they would rather endure traffic congestion than pay a five-dollar-per day fee.——能不能影响到交通堵塞?无关~pass
(E) During the average workday, private vehicles owned and operated by people living within the city account for twenty percent of the city's traffic congestion.——能不能影响到交通堵塞?无关~pass

60.    26S
Because no employee wants to be associated with bad news in the eyes of a superior, information about serious problems at lower levels is progressively softened and distorted as it goes up each step in the management hierarchy.  The chief executive is, therefore, less well informed about problems at lower levels than are his or her subordinates at those levels.

The conclusion drawn above is based on the assumption that
雇员不希望自己的上司听到坏消息
所以坏的消息都被下级隐瞒了(逐级递增)
因此,CEO是听不到这些消息的(比其他的下属们)
问假设:CEO的消息来源是否只有下属一个?

(A) problems should be solved at the level in the management hierarchy at which they occur——无关啊
(B) employees should be rewarded for accurately reporting problems to their superiors——这是属于这样做的weaken嘛
(C) problem-solving ability is more important at higher levels than it is at lower levels of the management hierarchy——这也是属于这样做的weaken嘛
(D) chief executives obtain information about problems at lower levels from no source other than their subordinates——bingo!补齐了hole
(E) some employees are more concerned about truth than about the way they are perceived by their superiors ——这算是走题吧
作者: cyndichiang    时间: 2014-4-14 13:14
精炼题:
1. 25‘’  
BG:: Most antidepressant drugs cause weight gain
premise:dieting can help reduce the amount of weight gained while taking such antidepressants
conclusion:some weight gain is unlikely to be preventable
Q:Must be true
prephrase: at least some people who take this drug still gain weight
(A) A physician should not prescribe any
antidepressant drug for a patient if that
patient is overweight.----"any" is incorrect ,since still some people will not gain weight
(B) People who are trying to lose weight should not
ask their doctors for an antidepressant drug.-----"should not" is incorrect ,some people will not gain weight
(C) At least some patients taking antidepressant
drugs gain weight as a result of taking them.-----Correct,some people can gain weight and some not .Some people can get gained weight reduced through dieting,but some people not.
(D) The weight gain experienced by patients taking
antidepressant drugs should be attributed to
lack of dieting.-----"should be attributed to "is incorrect,since some people are not affected by the drug
(E) All patients taking antidepressant drugs should
diet to maintain their weight.----"All" is incorrect ,since not all the people can gain weight by taking this drug

答案选对了,但是还是没有理解题目,看了答案才知道
reduce the amount of weight gained的意思是能够减少已经增加的的体重,也就是本来增加20 现在只增加10


逻辑链:
1.2.   33‘’
R: The alarming fact is that 90 percent of the people in this country now report that they know someone who is unemployed.
S: premise:But a normal, moderate level of unemployment is 5 percent
conclusion:?? Prephrase: S disagrees with R
Q1:must be true
Q2.Assumption of S' argument
premise:moderate level of unemployment is 5 percent, with 1 out of 20 workers unemployed
conclusion: if a person knows approximately 50 workers, 1 or more will very likely be unemployed
prephrase: the unemployment distribute evenly in this country.

剩下回来补~

作者: cyndichiang    时间: 2014-4-14 17:41
逻辑链
3. 23‘’
premise: the fee will exceed the cost of round-trip bus fare from many nearby points
conclusion:many people will switch from using their cars to using the bus
Q:Weaken;
prephrase: fee is not a stimulus for people to switch their behavior


4. 34''
premise: serious problems at lower levels is progressively softened as it goes up each step in the management hierarchy
conclusion: CEO is less informed about the problems at lower level than his subordinates
Q;Assumption
Prephrase: CEO can only get information at lower level from his subordinates
作者: simonleo    时间: 2014-5-9 09:46
1.
Time:25s
P:drug cause weight gain
eat less can help to keep fit
but still will gain some weight.
pre:must be true
(A) A physician should not prescribe any
antidepressant drug for a patient if that
patient is overweight. 从原文信息得不出
(B) People who are trying to lose weight should not
ask their doctors for an antidepressant drug. 正解,因为some weight gain is unlikely to be preventable.
(C) At least some patients taking antidepressant
drugs gain weight as a result of taking them. 不是some吧,是几乎都会gain weight吧
(D) The weight gain experienced by patients taking
antidepressant drugs should be attributed to
lack of dieting. 错
(E) All patients taking antidepressant drugs should
diet to maintain their weight. 没有此建议
1 B 错 应选C B是exaggerate项
2.44s A
3.17s C 错 应选B C是结论,B取反以后 说失业人群一般集中在地理位置都很分散的地方,即深层含义是他们互相认识的人都不太一样 即可以削弱原文的互相认识的这个前提。
4.40s B
5.36s D

作者: cechymery    时间: 2014-8-1 17:15
A选项对于这个drug不开给超重的人还有可能是因为别的原因,不一定会加强,B选项同样如此,有可能是因为增重的效果,有可能是因为这个药会产生其他的副作用所以选C
作者: spencerchan    时间: 2015-8-21 11:23
24’’
B: Drugs lead weight gain
P: Dieting can help, but some weight gain is unavoidable
Prep: C: Drugs would lead weight gain
A: Any antidepressant is incorrect
B: ??
C: CORRECT
D: Wrong
E: All patient

1’20’’
P: Normally, 1 out of 20 workers unemployed; if people know 50 people, then 1 or more will likely be unemployed
Prep: C: repetitive???
A: CORRECT
B: Did not mention it
C: Wrong
D: Wrong
E: Wrong

11’’
P: BUT, normally 1 out of 20 workers unemployed; if people know 50 people, then 1 or more will likely un employed
Prep: Repetitive??
A: Irrelevant
B: CORRECT
C: Irrelevant
D: Not a assumption
E: Irrelevant

30’’
P: a fee on private vehicles entering the city
C: People will switch from using cars to using the bus
Prep: People will till use cars even a fee needed => More convenient? bus is also costly?
A: Strengthen
B: CORRECT
C: Irrelevant
D: Irrelevant
E: Strengthen

27’’
P: Subordinators would not report bad news to managers
C: Managers would less well informed about problems
Prep: There are other ways that managers get information
A: Irrelevant
B: Weaken
C: Irrelevant
D: CORRECT
E: Irrelevant => way, truth

今天的题也做到过几道,所以又全对啦~哈哈哈,还是开心的
作者: spencerchan    时间: 2015-8-21 11:24
24’’
B: Drugs lead weight gain
P: Dieting can help, but some weight gain is unavoidable
Prep: C: Drugs would lead weight gain
A: Any antidepressant is incorrect
B: ??
C: CORRECT
D: Wrong
E: All patient

1’20’’
P: Normally, 1 out of 20 workers unemployed; if people know 50 people, then 1 or more will likely be unemployed
Prep: C: repetitive???
A: CORRECT
B: Did not mention it
C: Wrong
D: Wrong
E: Wrong

11’’
P: BUT, normally 1 out of 20 workers unemployed; if people know 50 people, then 1 or more will likely un employed
Prep: Repetitive??
A: Irrelevant
B: CORRECT
C: Irrelevant
D: Not a assumption
E: Irrelevant

30’’
P: a fee on private vehicles entering the city
C: People will switch from using cars to using the bus
Prep: People will till use cars even a fee needed => More convenient? bus is also costly?
A: Strengthen
B: CORRECT
C: Irrelevant
D: Irrelevant
E: Strengthen

27’’
P: Subordinators would not report bad news to managers
C: Managers would less well informed about problems
Prep: There are other ways that managers get information
A: Irrelevant
B: Weaken
C: Irrelevant
D: CORRECT
E: Irrelevant => way, truth

今天的题也做到过几道,所以又全对啦~哈哈哈,还是开心的
作者: 千晤meli    时间: 2015-8-26 11:34
马克下第三题。

Roland:  The alarming fact is that 90 percent of the people in this country now report that they know someone who is unemployed.

Sharon:  But a normal, moderate level of unemployment is 5 percent, with 1 out of 20 workers unemployed.  So at any given time if a person knows approximately 50 workers, 1 or more will very likely be unemployed.

Sharon's argument relies on the assumption that

(A) normal levels of unemployment are rarely exceeded
      --------无关,normally level 只是举例说明
(B) unemployment is not normally concentrated in geographically isolated segments of the population
       -------说明S的论点是普遍现象而不是因为地理原因集中的表现,加强S的论点。
(C) the number of people who each know someone who is unemployed is always higher than 90% of the population
(D) Roland is not consciously distorting the statistics he presents
        -------做题时看错了,以为是说S的观点数据可信呢..
(E) knowledge that a personal acquaintance is unemployed generates more fear of losing one's job than does knowledge of unemployment statistics

作者: MIA926    时间: 2015-9-8 11:01
20150908 CR小分队
精练

2. Most antidepressant drugs cause weight gain. While
dieting can help reduce the amount of weight gained
while taking such antidepressants, some weight gain
is unlikely to be preventable.
The information above most strongly supports which
one of the following? ---Must be true
计时:16.94
逻辑链:P1:抗抑郁药会导致体重增加;P2:吃抗抑郁药时节食可以减少增加体重,但还是会长些肉
推测:节食可能导致更抑郁了然后增加药量然后体重增加的更多

(A) A physician should not prescribe any
antidepressant drug for a patient if that
patient is overweight. 新信息
(B) People who are trying to lose weight should not
ask their doctors for an antidepressant drug. 正确
(C) At least some patients taking antidepressant
drugs gain weight as a result of taking them.新信息
(D) The weight gain experienced by patients taking
antidepressant drugs should be attributed to
lack of dieting. 新信息
(E) All patients taking antidepressant drugs should
diet to maintain their weight. 新信息


PREP 练习
57.    (34655-!-item-!-188;#058&007580)
Roland:  The alarming fact is that 90 percent of the people in this country now report that they know someone who is unemployed.
Sharon:  But a normal, moderate level of unemployment is 5 percent, with 1 out of 20 workers unemployed.  So at any given time if a person knows approximately 50 workers, 1 or more will very likely be unemployed.
Sharon's argument is structured to lead to which of the following as a conclusion?---Must be true
逻辑链:R:这个国家90%的人都有认识失业的 人
S:正常的失业率是5%,只要一个人认识大约50个人,这其中就可能有不止一个失业的人
推测:这个国家的失业率不高

(A) The fact that 90% of the people know someone who is unemployed is not an indication that unemployment is abnormally high. 正确
(B) The current level of unemployment is not moderate. 新信息
(C) If at least 5% of workers are unemployed, the result of questioning a representative group of people cannot be the percentage Roland cites. 无关
(D) It is unlikely that the people whose statements Roland cites are giving accurate reports. 错误
(E) If an unemployment figure is given as a certain percent, the actual percentage of those without jobs is even higher. 新信息


58.    (34705-!-item-!-188;#058&007581)
Roland:  The alarming fact is that 90 percent of the people in this country now report that they know someone who is unemployed.
Sharon:  But a normal, moderate level of unemployment is 5 percent, with 1 out of 20 workers unemployed.  So at any given time if a person knows approximately 50 workers, 1 or more will very likely be unemployed.
Sharon's argument relies on the assumption that---Assumption
(A) normal levels of unemployment are rarely exceeded 无关
(B) unemployment is not normally concentrated in geographically isolated segments of the population 正确
(C) the number of people who each know someone who is unemployed is always higher than 90% of the population 无关
(D) Roland is not consciously distorting the statistics he presents 无关
(E) knowledge that a personal acquaintance is unemployed generates more fear of losing one's job than does knowledge of unemployment statistics 无关


59.    (34753-!-item-!-188;#058&007583)
A certain mayor has proposed a fee of five dollars per day on private vehicles entering the city, claiming that the fee will alleviate the city's traffic congestion.  The mayor reasons that, since the fee will exceed the cost of round-trip bus fare from many nearby points, many people will switch from using their cars to using the bus.
Which of the following statements, if true, provides the best evidence that the mayor's reasoning is flawed?---Weaken
逻辑链:市长说对进入这个城市的私家车收费就会减少交通堵塞,因为收的费用会大于bus的费用,所以很多人会改用bus
推测:这个基于了一个无端假设,就是人们会把费用作为选择出行工具的首要考量,但是很有可能很多人进城只能开车,没有公交路线经过他们的住所,所以这个方案达不到想要的结果

(A) Projected increases in the price of gasoline will increase the cost of taking a private vehicle into the city. Cost无关
(B) The cost of parking fees already makes it considerably more expensive for most people to take a private vehicle into the city than to take a bus. 微削弱 但是基本无关
(C) Most of the people currently riding the bus do not own private vehicles. 无关 讨论的是现在有车的人
(D) Many commuters opposing the mayor's plan have indicated that they would rather endure traffic congestion than pay a five-dollar-per day fee.  正确
(E) During the average workday, private vehicles owned and operated by people living within the city account for twenty percent of the city's traffic congestion. 加强原观点


60.    (34801-!-item-!-188;#058&007584)
Because no employee wants to be associated with bad news in the eyes of a superior, information about serious problems at lower levels is progressively softened and distorted as it goes up each step in the management hierarchy.  The chief executive is, therefore, less well informed about problems at lower levels than are his or her subordinates at those levels.
The conclusion drawn above is based on the assumption that---Assumption
逻辑链:下级的员工不想跟坏消息扯上关系所以在向上传达消息的过程中总是会歪曲消息,所以经理总是不能很好地得知员工的情况
推测:经理只能通过下级员工得知消息
(A) problems should be solved at the level in the management hierarchy at which they occur 无关
(B) employees should be rewarded for accurately reporting problems to their superiors 无关
(C) problem-solving ability is more important at higher levels than it is at lower levels of the  management hierarchy 无关
(D) chief executives obtain information about problems at lower levels from no source other than their subordinates 正确
(E) some employees are more concerned about truth than about the way they are perceived by their superiors  加强


作者: 虫虫爱小葱    时间: 2015-9-10 17:02
2.P:大部分anti会导致体重上升,虽然饮食可以控制一些由anti导致的体重,但是还有一些体重上升是没办法避免的。
must be true.
推断:在控制饮食的情况下,食用anti还是会导致轻微体重上升  C
57.Fact:R:需要注意的是有90&的人都说他们知道有人失业
P:S:正常失业率是5%,20个人里会有一个。所以只要一个人认识50个人,其中一个或多个可能失业
must be true
C:R说的事实是正常现象     A
58.assumption    ??C
59.
fact:mayor会对开私家车的人收5刀
P:这个收费远高于乘坐公交车的费用,所以人们将会由开车转变成坐公交车
C:能有效缓解交通拥堵
weaken--plan to goal
推断:人们可能有第三种交通方式;公交车数量的增加会增加交通拥挤;  D方案无法实施
60
P:没有雇员想让上级把自己和坏消息联系起来,所以消息在逐层上传的过程中被弱化和扭曲。
C:chief知道的消息严重程度要比他的下属低
assumption
推断:chief没有别的渠道获得信息--如果有的话,那么雇员对消息的弱化也无法影响chief  D
作者: 虫虫爱小葱    时间: 2015-9-10 17:19
自己mark一下
57问的是通过S的话能得出来什么结论。与90%的人无关。何况C本身也对题目没有影响。
B取非则不对
作者: elva85    时间: 2015-9-11 00:35
练习方法如下:
1. 计时看问题和题干,一般在45s内。
2. 先不看选项、不看题干按自己的理解写出逻辑链。
3. 合理推测正确选项的可能方向
4. 看选项,选出正确答案

用时 15秒
c:anti... drugs cause weight gained
P: drugs抑制reduce weight gain
predict:
举一些人吃了药,另一些人没吃药的数据或例子
C

time:43.5
b:一个城市90%的人认识一些失业的人
P:5%的人失业,1个失业比20个就业的人
C: if a person know 50 peoples,1 or more likely be unemployed
predict:
A

time:
同上
B

time:55
b:公交价格每人5元
p:这个费用超过在公交很多点来回的费用
c:大家都用自己的车-路上车多导致拥挤
predict:
如果有车的人很少?多少数量的车才会导致拥挤?油价高于乘坐公交的成本?
c

time:29
b:下属不希望在领导眼里不好
p:不提供或歪曲不好的信息给上级
c:主管从底层员工得知的问题的信息少于他下属
predict:
有没有其他的渠道主管可以知道这个信息?
D
作者: elva85    时间: 2015-9-11 00:40
错因分析:
第四题审题不仔细,说的是证明那人的推测是错的,选C变成了加强。
作者: fesche    时间: 2016-5-14 01:01
drug —> weight gain
diet —> prevent weight gain
some weight gain is unlikely to be preventable
not preventable
support following
B-C
(A) A physician should not prescribe any antidepressant drug for a patient if that patient is overweight.
(B) People who are trying to lose weight should not ask their doctors for an antidepressant drug.
(C) At least some patients taking antidepressant drugs gain weight as a result of taking them.
(D) The weight gain experienced by patients taking antidepressant drugs should be attributed to lack of dieting.
(E) All patients taking antidepressant drugs should diet to maintain their weight.

90% know one unemployed
unemployment in the country is 5%
1 out of 50 is likely to be unemployed
conclusion
90% people know more than 50 workers?
A
(A) The fact that 90% of the people know someone who is unemployed is not an indication that unemployment is abnormally high.
(B) The current level of unemployment is not moderate.
(C) If at least 5% of workers are unemployed, the result of questioning a representative group of people cannot be the percentage Roland cites.
(D) It is unlikely that the people whose statements Roland cites are giving accurate reports.
(E) If an unemployment figure is given as a certain percent, the actual percentage of those without jobs is even higher.

90% know one unemployed
unemployment in the country is 5%
1 out of 50 is likely to be unemployed
assumption
people know more than 50 workers
B
(A) normal levels of unemployment are rarely exceeded
(B) unemployment is not normally concentrated in geographically isolated segments of the population
(C) the number of people who each know someone who is unemployed is always higher than 90% of the population
(D) Roland is not consciously distorting the statistics he presents
(E) knowledge that a personal acquaintance is unemployed generates more fear of losing one's job than does knowledge of unemployment statistics

fee, reduce congestion
entering fee exceeds bus
C: take bus from car       
flaw,不能得出不开车
住的附近没有bus,bus运营时间不对
C-B
(A) Projected increases in the price of gasoline will increase the cost of taking a private vehicle into the city.
(B) The cost of parking fees already makes it considerably more expensive for most people to take a private vehicle into the city than to take a bus.
(C) Most of the people currently riding the bus do not own private vehicles.
(D) Many commuters opposing the mayor's plan have indicated that they would rather endure traffic congestion than pay a five-dollar-per day fee.
(E) During the average workday, private vehicles owned and operated by people living within the city account for twenty percent of the city's traffic congestion.

employees tend to give soften report to their supervisor
C, CEO is less well informed
Assumption
GAP:每个人都会报告得轻一点
D
(A) problems should be solved at the level in the management hierarchy at which they occur
(B) employees should be rewarded for accurately reporting problems to their superiors
(C) problem-solving ability is more important at higher levels than it is at lower levels of the management hierarchy
(D) chief executives obtain information about problems at lower levels from no source other than their subordinates
(E) some employees are more concerned about truth than about the way they are perceived by their superiors

作者: 宋痂岛__    时间: 2017-12-18 21:33
CABBD
作者: May97    时间: 2018-2-2 12:54
T: 22s
P: Most antidepressant drugs cause weight gain.
P: While dieting can help reduce the amount of weight gained
P: while taking such antidepressants, some weight gain is unlikely to be preventable.
Conclusion:
(A) A physician should not prescribe any antidepressant drug for a patient if that patient is overweight.
(B) People who are trying to lose weight should not ask their doctors for an antidepressant drug.
(C) At least some patients taking antidepressant drugs gain weight as a result of taking them.--->correct
(D) The weight gain experienced by patients taking antidepressant drugs should be attributed to lack of dieting.
(E) All patients taking antidepressant drugs should diet to maintain their weight.

T: 34s
P: 90 percent of the people in this country now report that they know someone who is unemployed.
P: moderate level of unemployment is 5 percent, with 1 out of 20 workers unemployed.  
P: So at any given time if a person knows approximately 50 workers, 1 or more will very likely be unemployed.
Conclusion:
(A) The fact that 90% of the people know someone who is unemployed is not an indication that unemployment is abnormally high.--->CORRECT
(B) The current level of unemployment is not moderate.
(C) If at least 5% of workers are unemployed, the result of questioning a representative group of people cannot be the percentage Roland cites.
(D) It is unlikely that the people whose statements Roland cites are giving accurate reports.
(E) If an unemployment figure is given as a certain percent, the actual percentage of those without jobs is even higher.--->correct

FA:
(A) normal levels of unemployment are rarely exceeded
(B) unemployment is not normally concentrated in geographically isolated segments of the population--->CORRECT???
(C) the number of people who each know someone who is unemployed is always higher than 90% of the population
(D) Roland is not consciously distorting the statistics he presents
(E) knowledge that a personal acquaintance is unemployed generates more fear of losing one's job than does knowledge of unemployment statistics--->CORRECT
前后两题搞不清楚差别了。。。
T: 32S
P: A certain mayor has proposed a fee of five dollars per day on private vehicles entering the city,
C: claiming that the fee will alleviate the city's traffic congestion.  
P: The mayor reasons that, since the fee will exceed the cost of round-trip bus fare from many nearby points, many people will switch from using their cars to using the bus.
FLAW:
(A) Projected increases in the price of gasoline will increase the cost of taking a private vehicle into the city.
(B) The cost of parking fees already makes it considerably more expensive for most people to take a private vehicle into the city than to take a bus.--->CORRECT
(C) Most of the people currently riding the bus do not own private vehicles.
(D) Many commuters opposing the mayor's plan have indicated that they would rather endure traffic congestion than pay a five-dollar-per day fee.
(E) During the average workday, private vehicles owned and operated by people living within the city account for twenty percent of the city's traffic congestion.

T: 34S
P: Because no employee wants to be associated with bad news in the eyes of a superior, information about serious problems at lower levels is progressively softened and distorted as it goes up each step in the management hierarchy.  
C: The chief executive is, therefore, less well informed about problems at lower levels than are his or her subordinates at those levels.
FA:
(A) problems should be solved at the level in the management hierarchy at which they occur
(B) employees should be rewarded for accurately reporting problems to their superiors
(C) problem-solving ability is more important at higher levels than it is at lower levels of the management hierarchy
(D) chief executives obtain information about problems at lower levels from no source other than their subordinates--->CORRECT
(E) some employees are more concerned about truth than about the way they are perceived by their superiors
作者: echo-LUO    时间: 2018-9-25 15:06
精练
药物导致体重增加——>节食可以减少体重——>但是服药体重增加不可避免
A:饮食减少的小于药物增加的;服药期间不能饮食减重
(A) A physician should not prescribe any antidepressant drug for a patient if that patient is overweight. 无关
(B) People who are trying to lose weight should not ask their doctors for an antidepressant drug. 想减肥的不应该吃药,似乎有关联
(C) At least some patients taking antidepressant drugs gain weight as a result of taking them. “至少”,无关
(D) The weight gain experienced by patients taking antidepressant drugs should be attributed to lack of dieting. 吃药体重增加是缺乏节食引起的——>节食会保持体重——>和题目不符
(E) All patients taking antidepressant drugs should diet to maintain their weight. 无

错误分析:
题目没认真看,问的是,题干的内容可以支持哪个选项
B选项其实已经犯了大忌,“should not”这种表述,很有可能是错的。楼上分析的很有道理,万一这些减肥的人只能服用这个药物呢?
C选项就是一个现象,一些患者会因为吃药体重增加,和题目一致

逻辑链
57.
R:90%的人都认识至少一个失业者
S:一般失业率是5%,一个人至少认识超过50个人,就能认识1个以上的失业者
(A) The fact that 90% of the people know someone who is unemployed is not an indication that unemployment is abnormally high. ——>很完美,90%不能说明失业率高
(B) The current level of unemployment is not moderate. 相反
(C) If at least 5% of workers are unemployed, the result of questioning a representative group of people cannot be the percentage Roland cites. 无关
(D) It is unlikely that the people whose statements Roland cites are giving accurate reports. 无关
(E) If an unemployment figure is given as a certain percent, the actual percentage of those without jobs is even higher. 无关



58.
题目相同,问假设
(A) normal levels of unemployment are rarely exceeded 无关
(B) unemployment is not normally concentrated in geographically isolated segments of the population——>取反,如果是集中在地理位置分割的地方,很可能认识的50个人都不是失业的人,保留
(C) the number of people who each know someone who is unemployed is always higher than 90% of the population ——>取反,没影响吧
(D) Roland is not consciously distorting the statistics he presents 无关
(E) knowledge that a personal acquaintance is unemployed generates more fear of losing one's job than does knowledge of unemployment statistics 无关



59.
mayor认为提高车辆进入费用,会缓解拥堵——>费用高于坐公交车——>大家会纷纷不开私家车,去坐公交
A:可能一开始自己开车的钱就比坐公交车贵(油费啊啥的)
(A) Projected increases in the price of gasoline will increase the cost of taking a private vehicle into the city. 加强了
(B) The cost of parking fees already makes it considerably more expensive for most people to take a private vehicle into the city than to take a bus. !!和我猜的一样
(C) Most of the people currently riding the bus do not own private vehicles. 无关
(D) Many commuters opposing the mayor's plan have indicated that they would rather endure traffic congestion than pay a five-dollar-per day fee. 无关
(E) During the average workday, private vehicles owned and operated by people living within the city account for twenty percent of the city's traffic congestion. 无关



60.
昨天刚做过,不重复做了









作者: 云栈    时间: 2019-11-5 04:22
1.        14s
P: 大多数抗抑郁药会导致体重增加。
P: 节食可以帮助减少服用此类抗抑郁药时增加的体重,但不太可能防止体重增加。
推测:diet reduce的重量没有吃这种药增加的数量多
选项分析:C
A: 无关,题中没有提到overweight的危害之类的信息
B:无关,同A
C: 正确,paraphrase了第一句话
D: 错的,题上说了有的人diet了体重也会增加
E: 错误,同D

2.        26s
P:令人震惊的事实是,这个国家现在有90%的人报告说他们认识某个失业的人。
P: 但正常的中等失业率是5%,每20名工人中就有1名失业。
C: 因此,在任何给定时间,如果一个人认识大约50名工人,那么很可能会有1个或更多的工人失业。
推测:并不是说有很多人都认识失业的人,就说明失业率很高
选项分析:A

3.        26s
P:令人震惊的事实是,这个国家现在有90%的人报告说他们认识某个失业的人。
P: 但正常的中等失业率是5%,每20名工人中就有1名失业。
C: 因此,在任何给定时间,如果一个人认识大约50名工人,那么很可能会有1个或更多的工人失业。
推测:每个人都认识各行各业的人;失业率不集中在某个行业
选项分析:B

4.        27s
P: 某市长提议对进入市区的私家车收取每天5美元的费用,声称该费用将减轻该市的交通拥堵。
C: 市长之所以这样说,是因为该费用将超过附近许多点的往返巴士票价,许多人将从使用汽车转向使用公共汽车。
推测:很多进入城市的人去的是一些bus无法到达的地方;他们进入城市都有必须要开车的原因
选项分析:B

5.        28s
P: 因为没有一个员工希望在上级主管的眼中与坏消息相关联,所以有关低层严重问题的信息随着在管理层次结构中的每个步骤上的应用而逐渐被软化和扭曲。
C: 因此,与较低级别的下属相比,首席执行官对较低级别的问题了解得更少。
推测:高管只能通过下级获取到基层员工获取的信息。
选项分析:D

作者: Lincy123    时间: 2020-5-13 22:20
药会增重,节食可以减少药物增重的量,但增重不大可能避免
Must be true
A) 如果病人超重,医生就不能给他开A药,
B) 想减肥的人不能向医生要A药
C) 至少有部分人因为吃药增重。Correct
D)吃药增重的原因是病人没节食。x
E) 所有病人都要节食维持体重。X

S:失业率5%,所以一个人认识50人,其中会有1人失业
A)90%的人认识一个失业的人并非不寻常的事情。Correct
B)现在的失业率不moderate。无关
C)如果5%的人失业,询问代表群体的结果不能成为百分比
D)谈话的人没有精确报告
E)失业率是确切百分比的话,真实数字会更高。

A) 失业的正常水平不会过量
B) 失业不是集中在人口的地理隔离。取非,失业集中在人口隔离地区,weaken。Correct
C) 认识一个失业的人的人的数量总是超过90%。无关
D)R没有有意曲解数据。取非,R有意曲解数据。x
E) 认识一个失业的人比知道失业率还要产生更多失业恐惧。无关

P:私家车进城费比公交车高
C:许多人会改坐公交而非开车
Weaken:大家依然开车的原因
A) 油价上涨会增加开车成本。Strengthen
B) 停车费贵增加开车成本。Strengthen x correct
既然停车早就比公交贵了,但人们还是选择公交,说明不是钱的原因,加进城费没用
C) 大部分搭公交的人没有车。无关
D)许多人宁可忍受交通堵塞也不要交5元。Correct x
E) 在平常,由城市内的私家车造成的堵塞占了20%。

P:当坏事从下往上传到每个管理层级都会被削弱
C:相较于直属下属,总经理很少从更低职位的人听到问题
Assumption:每一阶级只能从直属下属获得消息
A) 问题要在发生的管理阶级被解决。
B) 员工要因为准确向上级报告被奖励
C) 高层的问题解决能力比低层更重要、
D)高层只能从直属下属获得信息。Correct
E) 一些员工更关心事实






欢迎光临 ChaseDream (https://forum.chasedream.com/) Powered by Discuz! X3.3